Test Bank – ACP: Nephrology

Renal Function and Disorders of Water and Sodium Balance

1. A 52-year-old woman presents to the emergency department after experiencing 4 days of worsening mental status. She has not seen a doctor in over 20 years. Her medical history is unremarkable. She takes no medications. Physical examination shows a somnolent, obese woman with dry mucous membranes. The neurologic examination is nonfocal. Results of laboratory studies are as follows: sodium concentration, 128 mEq/L; potassium concentration, 4.5 mEq/L; chloride concentration, 94 mEq/L; serum glucose level, 810 mg/dl; renal function, normal.

Which of the following would be the most appropriate intervention in the care of this patient?

A. Fluid restriction and loop diuretics
B. Administration of hypertonic saline
C. Administration of normal saline and insulin
D. Demeclocycline

Key Concept/Objective: To be able to recognize hyperglycemic hyponatremia

Hyperglycemia lowers the plasma sodium concentration; in the absence of insulin, glucose is an effective osmole that attracts water from cells and thereby dilutes extracellular sodium. Therefore, the blood glucose level should always be examined when a low plasma sodium concentration is being evaluated. The plasma sodium concentration falls by approximately 3 mEq for every 200 mg/dl (10 mmol) increase in blood glucose and will increase by this amount when hyperglycemia is corrected with insulin. To evaluate hyponatremia in the presence of hyperglycemia, the serum sodium concentration must be “corrected” for the osmotic effect of glucose. In this patient, the corrected sodium concentration is within normal limits, and the sodium value will normalize once the high glucose level is treated. Hypertonic fluid, fluid restriction, loop diuretics, and demeclocycline can all be used in the management of hyponatremia, depending on the etiology of the problem. (Answer: C—Administration of normal saline and insulin)

2. A 60-year-old man comes to your clinic for follow-up. He has no new complaints. He has a history of hypothyroidism and mild asthma. His medications include levothyroxine and albuterol. The patient has been smoking two packs of cigarettes a day for 30 years. His physical examination is unremarkable. Results of laboratory studies are as follows: sodium concentration, 126 mEq/L; potassium concentration,
3.8 mEq/L; chloride concentration, 96 mEq/L; bicarbonate level, 24 mEq/L; blood urea nitrogen (BUN),
6 mg/dl; glucose and creatinine levels, normal; serum osmolality, 258 mOsm/L; urinary sodium, 56 mEq/L; urinary osmolality, 360 mOsm/kg. Thyroid function tests are within normal limits.

Which of the following interventions is most likely to normalize this patient’s sodium level?

A. Increasing the dose of levothyroxine
B. Restricting fluid intake
C. Administering intravenous normal saline
D. Starting a thiazide

Key Concept/Objective: To understand the diagnosis and treatment of the syndrome of inappropriate antidiuretic hormone secretion (SIADH)

SIADH is a nonosmotic release of vasopressin in the absence of a hemodynamic stimulus to account for it. In SIADH, urinary sodium matches intake; as the urine is usually concentrated, the urinary sodium concentration exceeds 40 mEq/L unless dietary sodium intake is very low. The urinary osmolality also is inappropriately increased. BUN and serum uric acid are usually low in patients with SIADH. A number of disorders can be associated with SIADH. Tumors (most commonly small cell carcinoma of the lung) can ectopically synthesize and secrete vasopressin. Unexplained, persistent hyponatremia should be considered a marker for an underlying malignancy. SIADH is a mechanism for developing hyponatremia, not a diagnosis. In all patients with SIADH, a specific etiology for inappropriate vasopressin secretion should be sought. In a patient with clinical features of SIADH that has no obvious cause, a more extensive evaluation is indicated. The workup should include a careful search for malignancy and central nervous system pathology and an endocrine evaluation to exclude hypothyroidism and hypocortisolism. In patients with asymptomatic hyponatremia secondary to SIADH, the treatment of choice is fluid restriction. In this patient, there is no laboratory evidence of hypothyroidism, so increasing the dose of levothyroxine will not be helpful. Administration of normal saline in patients with SIADH can worsen the hyponatremia. Thiazides block the reabsorption of sodium and chloride in the distal tubule and can lead to severe hyponatremia. (Answer: B—Restricting fluid intake)

3. A 34-year-old woman presents to your clinic complaining of polyuria and polydipsia of 4 months’ duration. She says she is urinating between 20 to 30 times every day. Her medical history is unremarkable. She takes oral contraceptives and a multivitamin. Her family history is significant for diabetes and coronary artery disease. Physical examination is unremarkable. Her sodium concentration is 143 mEq/L; her potassium, creatinine, and glucose levels are normal. Urine osmolality is 160 mOsm/kg.

Which of the following is the most likely diagnosis for this patient?

A. Diabetes insipidus
B. Psychogenic polydipsia
C. Beer potomania
D. Salt poisoning

Key Concept/Objective: To be able to recognize diabetes insipidus

This patient has polyuria with diluted urine and a serum sodium level in the high normal range. A diagnosis of diabetes insipidus can be made if the urine osmolality is less than 250 mOsm/kg despite hypernatremia (a serum sodium level greater than 143 mEq/L). When the disease is suspected in a polyuric patient whose serum sodium concentration is normal, the urine osmolality can be monitored while the patient is deprived of water, allowing the serum sodium level to increase to 143 mEq/L. Exogenous vasopressin increases urine osmolality by more than 150 mOsm/kg in patients with neurogenic (but not nephrogenic) diabetes insipidus. It is possible to misdiagnose diabetes insipidus in patients who actually have a primary thirst disorder. Excessive water intake suppresses vasopressin secretion and causes polyuria with dilute urine. Because patients with primary polydipsia secrete vasopressin normally, they do not become hypernatremic during diagnostic water deprivation. Correlation with plasma vasopressin levels is often necessary in borderline cases. Diabetes insipidus can be classified as neurogenic or nephrogenic. Neurogenic diabetes insipidus is caused by deficient secretion of vasopressin; nephrogenic diabetes insipidus results from the kidney’s unresponsiveness to normally secreted hormone. With either disorder, patients present with polyuria and polydipsia. Most patients with polyuria do not become hypernatremic, because thirst maintains electrolyte-free water balance. Beer potomania will typically cause dilutional hyponatremia. Salt poisoning can cause hypernatremia with a high urinary osmolality. (Answer: A—Diabetes insipidus)

For more information, see Sterns RH: 10 Nephrology: I Renal Function and Disorders of Water and Sodium Balance. ACP Medicine Online (www.acpmedicine.com). Dale DC, Federman DD, Eds. WebMD Inc., New York, September 2003

Disorders of Acid-Base and Potassium Balance

4. A 22-year-old woman presents to the emergency department with nausea, vomiting, and abdominal pain of 4 days’ duration. Her fluid balance profile is as follows: Na+, 145; K+, 5.0; Cl-, 105; HCO3 , 15; BUN, 37; Cr, 1.6; glucose, 780; UA, 4+ ketones.

What is the best initial treatment of this patient’s acid-base disorder?

A. Free water
B. Normal saline
C. Normal saline, sodium bicarbonate, and insulin
D. Half-normal saline and insulin
E. Normal saline and insulin

Key Concept/Objective: To understand the diagnosis and treatment of diabetic ketoacidosis

Metabolic acidosis can be classified into two types: that associated with an elevation in the anion gap, and that in which the anion gap is normal. A calculation of the anion gap in this patient reveals a gap of 25. Among the causes of acidosis associated with an elevated anion gap are alcoholic ketoacidosis, lactic acidosis, starvation, ingestion of alcohols, ingestion of salicylates, and diabetic ketoacidosis. In patients with diabetic ketoacidosis (such as this patient), optimal initial treatment includes fluid replacement with normal saline to promote ketonuria and insulin to facilitate glucose transport. Bicarbonate therapy is not usually indicated unless the acidosis is severe or severe hyperkalemia is present. (Answer: E—Normal saline and insulin)

5. A 42-year-old woman presents with nausea, vomiting, and left flank pain with radiation to the groin; these symptoms have persisted for 3 days. A helical CT scan reveals a stone in the left ureter. On the basis of urinalysis and serum chemistries, a diagnosis of type 1 renal tubular acidosis (RTA) is made.

Which of the following is NOT consistent with type 1 RTA?

A. Normal-anion-gap metabolic acidosis
B. Urine pH < 5.3
C. Hypokalemia
D. Urinary calcium phosphate crystals
E. Sjögren syndrome

Key Concept/Objective: To understand the diagnosis of type 1 RTA

Renal tubular acidosis is one of the causes of normal-anion-gap metabolic acidosis. Other causes are administration of HCl and losses of bicarbonate from the gastrointestinal tract. Type 1 RTA may be congenital, or it may occur in association with various immune disorders, such as Sjögren syndrome. The underlying defect involves the inability of the intercalated cells of the collecting tubule to pump out hydrogen ions. As a result, the urine pH is always greater that 5.3. Hypokalemia occurs secondary to enhanced Na+-K+ exchange in the distal tubule, because hydrogen ions are not secreted in response to sodium reabsorption. A major complication of type 1 RTA is nephrocalcinosis. Nephrocalcinosis is caused by calcium phosphate crystals, which occur secondary to an increase in the resorption of proximal tubular citrate through metabolic acidosis. The decrease in urinary citrate facilitates the precipitation of calcium phosphate crystals in the collecting tubule. (Answer: B—Urine pH < 5.3)

6. A 68-year-old man with chronic renal insufficiency presents with weakness, paresthesias, and progressively worsening shortness of breath. He has been experiencing these symptoms for 4 days. Laboratory findings show a potassium level of 7.2; an electrocardiogram reveals peaked T waves and widening of the QRS complex.

Which of the following is NOT indicated in the initial treatment of this patient?

A. Intravenous calcium
B. Intravenous glucose and insulin
C. Dialysis
D. Sodium polystyrene sulfonate (Kayexalate, Kionex)
E. Beta blockers

Key Concept/Objective: To understand the diagnosis and treatment of hyperkalemia

The initial manifestations of hyperkalemia are usually neuromuscular in origin and are nonspecific. Diagnosis is based on serum potassium level; emergent treatment is based on whether cardiac arrhythmias are present or electrocardiographic changes are occurring. Treatment involves the use of intravenous calcium to reduce the excitability of cardiac cell membrane and use of intravenous glucose and insulin to facilitate transport of potassium into the intracellular space. Sodium polystyrene sulfonate is used to increase the excretion of potassium in the colon. In refractory cases, dialysis may be initiated to rapidly remove serum potassium. In addition, intranasal beta agonists may be used to transiently reduce serum potassium levels in the acute setting. Beta blockers can increase potassium levels. (Answer: E—Beta blockers)

7. A 35-year-old woman presents to your clinic with a history of headaches, weakness, fatigue, and polyuria. Her blood pressure is 210/94 mm Hg. Laboratory tests reveal the following abnormalities: arterial pH, 7.48; sodium, 148; potassium, 2.7; HCO -, 37; plasma renin level is low; urine chloride, 28 mEq/L.

Which of the following is the most likely diagnosis?

A. Secondary hyperaldosteronism
B. Diuretic abuse
C. Milk-alkali syndrome
D. Primary hyperaldosteronism
E. Type 4 RTA

Key Concept/Objective: To be able to differentiate between the various causes of metabolic alkalosis

Metabolic alkalosis is characterized by an elevation in serum bicarbonate level and a concomitant elevation of arterial pH. The causes of metabolic alkalosis include GI and renal losses of hydrogen ions; hypercalcemia; hypokalemia; excess alkali administration; and receiving thiazide or a loop diuretic. Primary hyperaldosteronism is characterized by hypertension, hypokalemia, hypernatremia, a low plasma renin level, and an elevated urine chloride level (as are seen in this patient). Secondary hyperaldosteronism is not usually associated with hypokalemia or metabolic alkalosis; it is usually associated with a high plasma renin level. Patients who have been abusing diuretics or who have milk-alkali syndrome often present with the serum values seen in this patient. However, in such patients, the urine chloride level is usually low (< 10), and these patients often present with volume depletion, whereas mild volume expansion is characteristic of primary hyperaldosteronism. Type IV RTA causes a metabolic acidosis. (Answer: D—Primary hyperaldosteronism)

For more information, see Black RM: 10 Nephrology: II Disorders of Acid-Base and Potassium Balance. ACP Medicine Online (www.acpmedicine.com). Dale DC, Federman DD, Eds. WebMD Inc., New York, December 2001

Approach to the Patient with Renal Disease

8. A 77-year-old man presents to the emergency department complaining that for the past 24 hours, he has been unable to void. The patient reports that he has had this problem intermittently and that he has been diagnosed with benign prostatic hyperplasia. He has an upper respiratory infection and has been taking over-the-counter pseudoephedrine. The patient also reports that a year ago, he was admitted to the hospital for unstable angina. At that time, his creatinine level was 1.1 mg/dl. In the emergency department, the patient’s serum creatinine level is determined to be 4.2 mg/dl.

Regarding this patient’s renal failure, which of the following statements is false?

A. Renal ultrasound should show bilateral hydronephrosis, suggesting obstructive uropathy as the cause of acute renal failure
B. The use of pseudoephedrine has contributed to the development of urinary obstruction
C. Urinalysis is commonly abnormal in obstructive uropathy; an abnormal result supports the diagnosis of obstruction
D. Obstruction impairs the ability of the kidneys to concentrate the urine and thus contributes to a polyuric state
E. Common causes of obstruction include nephrolithiasis and neurogenic bladder (and, in women, an enlarging cervical cancer)

Key Concept/Objective: To be able to recognize urinary obstruction as a cause of acute renal failure

Obstruction of urine flow can occur anywhere along the urinary tract, from the renal pelvis to the urethra. Anuria suggests complete urinary obstruction, although anuria can also be a feature of bilateral renal artery thrombosis, acute cortical necrosis, or severe acute tubular necrosis. In the absence of complete obstruction, urine flow may not necessarily be decreased and in fact is often increased. Chronic partial obstruction of the ureters leads to ureteral dilatation, which overcomes the blockage of urine flow. In addition, obstruction impairs the urinary concentrating ability and thus contributes to a polyuric state. Common causes of urinary obstruction include nephrolithiasis, prostate enlargement in men, neurogenic bladder in diabetic patients, and an enlarging cervical cancer in women. The urinalysis result is typically unremarkable in obstructive uropathy. The diagnosis is most often made by demonstrating ureteral dilatation on renal sonography. (Answer: C—Urinalysis is commonly abnormal in obstructive uropathy; an abnormal result supports the diagnosis of obstruction)

9. A 75-year-old woman with diabetes and hypertension is admitted to the hospital with nausea, vomiting, and abdominal pain. At admission, laboratory values include a blood urea nitrogen (BUN) measurement of 18 and a plasma creatinine measurement of 0.5 mg/dl. As part of her workup, she undergoes a contrast-enhanced CT scan of the abdomen. During the first 48 hours of the hospital stay, repeat laboratory studies reveal a plasma creatinine level of 1.1 mg/dl.

Which of the following statements is false regarding estimates of this patient’s glomerular filtration rate (GFR)?
A. A low baseline plasma creatinine value may lead to an overestimation of GFR because of decreased muscle mass in this elderly patient
B. The change in plasma creatinine level of 0.6 mg/dl represents an approximately 50% decrease in the patient’s GFR
C. The use of drugs such as cimetidine and trimethoprim increase plasma creatinine levels without affecting true GFR because of inhibition of tubular secretion of creatinine
D. The patient’s ideal body weight correlates inversely with GFR

Key Concept/Objective: To understand the use of creatinine clearance to estimate GFR and common factors that affect creatinine clearance

GFR is generally thought to be the best measure of renal function because filtration capacity correlates directly with the function of the nephron. Creatinine is produced at a relatively constant rate by hepatic conversion of skeletal muscle creatine, and the clearance of creatinine is used as an estimate of filtration. Creatinine clearance (CCr) can be measured by 24-hour collection of urine, or it can be estimated through use of a formula that involves the patient’s age, ideal body weight (IBW), and plasma creatinine (Cr): CCr = (140 – age) × (IBW in kg)/(72 × Cr). In females, the results are multiplied by a correction factor of 0.85. Because the production of creatinine is dependent on muscle mass, plasma levels are typically lower in elderly patients and in patients with conditions that result in profound muscle wasting. From the equation for estimating creatinine, it can be seen that creatinine clearance and GFR correlate directly with patient’s ideal body weight. Additionally, because creatinine clearance correlates inversely with plasma creatinine levels, a doubling of the plasma creatinine value (as seen in this patient) reflects a reduction in creatinine clearance by about half. It is important to note that creatinine clearance is not a perfect reflection of GFR because creatinine, in addition to being filtered, is also secreted in the tubules. Certain drugs, such as cimetidine, trimethoprim, and probenecid, inhibit this tubular secretion; this results in a decrease in creatinine clearance, but this decrease has no bearing on GFR. (Answer: D— The patient’s ideal body weight correlates inversely with GFR)

10. A 60-year-old man who presented with fatigue and bone pain is found to be anemic and thrombocytopenic. Examination reveals pale conjunctivae and thigh tenderness but no peripheral edema. Radiographs demonstrate osteolytic lesions in several thoracic vertebrae and the left femur. Serum chemistries reveal a creatinine level of 1.2 mg/dl, a calcium level of 9.5 mg/dl, a total protein level of 11 mg/dl, and an albumin level of 3.2 mg/dl. On bone marrow biopsy, there is replacement of normal marrow with sheets of plasma cells. Urinalysis is unremarkable, but a 24-hour urine study reveals proteinuria of 2.0 g/day.

Which of the following statements regarding this patient’s proteinuria is true?
A. The likely underlying pathology involves a structural abnormality
of the filtration barrier that results in loss of negatively charged proteins
B. It is likely that the results of the 24-hour urine study are falsely elevated because the urine dipstick test is sensitive for most protein species, including albumin and paraproteins
C. The proteinuria reflects an overproduction of normally filtered proteins, which overwhelms the reabsorptive capacity of the tubules
D. The patient’s degree of proteinuria and spectrum of clinical findings is consistent with the nephrotic syndrome

Key Concept/Objective: To understand the mechanisms of proteinuria and the means of detecting it

This patient has multiple myeloma, which is associated with an elevation in total protein level consistent with the production of immunoglobulin light chain paraproteins (Bence-Jones proteins). These positively charged proteins are filtered in large quantities and overwhelm the reabsorptive capacity of the tubules, resulting in significant proteinuria (overflow proteinuria). This finding is in contrast to those seen in conditions such as membranous nephropathy, diabetes mellitus, or minimal change disease; in these disorders, there is an alteration in the filtration barrier of the glomerulus, which allows negatively charged proteins such as albumin to be filtered excessively (glomerular proteinuria). These conditions can lead to the nephrotic syndrome, which is characterized by heavy proteinuria (> 3.5g/day) and concomitant hypoalbuminemia, edema, and hyperlipidemia. Results of standard dipstick testing will be positive only with negatively charged proteins such as albumin: thus, dipstick testing is insensitive with regard to the detection of Bence-Jones proteins, which are often positively charged. (Answer: C—The proteinuria reflects an overproduction of normally filtered proteins, which overwhelms the reabsorptive capacity of the tubules)

11. A 28-year-old man presents to clinic with a complaint of hematuria of 1 day’s duration. He was well until
3 days ago, when he developed a sore throat, low-grade fever, and malaise, which lasted for approximately 48 hours. He had a similar episode approximately 1 year ago. He denies having rash, joint pains, or dysuria. On examination, he appears well, and he is afebrile. His blood pressure is 118/62 mm Hg. Urine dipstick assay is significant for 2+ blood and trace protein. Microscopic examination of the urine reveals
10 to 15 red cells per high-powered field; dysmorphic red cells and occasional red cell casts are noted as well. Further testing reveals a normal antistreptolysin-O (ASO) titer and serum complement level.

Which of the following statements regarding this patient’s condition is false?
A. Findings on urinalysis identify the source of bleeding as glomerular in origin
B. Renal biopsy is likely to reveal mesangial deposition of immunoglobulin A (IgA) on immunofluorescence microscopy
C. Results of analysis of the urine sediment are consistent with a finding of hypercalciuria as a cause of the hematuria
D. The time course of the illness and the serum complement level help to differentiate this patient’s condition from acute postinfectious glomerulonephritis

Key Concept/Objective: To recognize Berger disease and the urinary findings associated with glomerulonephritis
Microscopic evaluation of urinary sediment is an important component of the workup of hematuria because it may help localize the bleeding to either the upper urinary tract (i.e., glomeruli) or the lower urinary tract (i.e., renal pelvis, ureters, bladder, urethra). The presence of red cell casts (formed from erythrocytes passing through the renal tubules) is virtually pathognomonic for acute glomerulonephritis. Dysmorphic red cells and red cell casts would not typically be seen in patients with hematuria caused by abnormalities of the lower urinary tract, such as nephrolithiasis, malignancy, or prostatitis. This patient presented with recurrent episodes of macroscopic hematuria following an upper respiratory infection, which is a common finding in patients with IgA nephropathy (Berger disease). This condition constitutes 10% to 40% of all cases of primary glomerulonephritis and is associated with increased serum IgA levels and mesangial deposition of IgA. Onset is typically in the second and third decades of life; although the disease typically progresses slowly, approximately one half of patients with IgA nephropathy will develop end-stage renal disease within 25 years of onset. Hypertension and proteinuria are predictive of more rapid disease progression. Serum complement levels are typically normal: a finding that helps to differentiate Berger disease from other causes of acute glomerulonephritis, such as acute postinfectious glomerulonephritis and lupus, in which complement levels are typically low. (Answer: C—Results of analysis of the urine sediment are consistent with a finding of hypercalciuria as a cause of the hematuria)

12. A 45-year-old man with a history of alcohol abuse is brought to the emergency department after being found lying on the floor of his apartment by a neighbor. On examination, he is unresponsive and appears dehydrated. Initial blood work reveals a hematocrit of 41%, a BUN of 60 mg/dl, and a creatinine level of 3.2 mg/dl. Creatine kinase and lactose dehydrogenase (LDH) levels are elevated at 12,000 U/L and 475 U/L, respectively. Urinalysis shows reddish urine with a specific gravity of 1.020; dipstick assay shows 3+ blood and 1+ protein. Microscopic examination of urine sediment demonstrates 0 to 2 red cells and 0 to 5 white cells per high-powered field; hyaline casts are also observed.

The urinary findings are suggestive of which of the following conditions?
A. Acute glomerulonephritis
B. Hemoglobinuria
C. Allergic interstitial nephritis
D. Myoglobinuria

Key Concept/Objective: To recognize the findings associated with myoglobinuria

This patient presents with rhabdomyolysis related to alcohol intoxication and prolonged immobilization. Myoglobin released from the breakdown of skeletal muscle is an endogenous nephrotoxin that can induce acute renal failure (ARF) by direct injury to tubular epithelial cells. ARF is a complication in up to one third of patients with rhabdomyolysis; factors that predispose to ARF in this setting include hypovolemia and acidosis. The prompt recognition of myoglobinuria is thus of paramount importance in this clinical setting and can be aided greatly by careful examination of the urine. Both myoglobin and hemoglobin (released from the breakdown of red cells in hemolytic processes) will react with the urine dipstick test for blood. The presence of pigments in the urine should be suspected when the results of dipstick testing are strongly positive for blood in the absence of red cells on microscopic examination. Acute glomerulonephritis is characterized by the finding of red cells and red cell casts on urinalysis. Acute (allergic) interstitial nephritis is suggested by the presence of white cell casts and nonpigmented granular casts. Eosinophiluria is an additional finding that suggests interstitial nephritis, though a finding of eosinophiluria is not highly sensitive. (Answer: D—Myoglobinuria)

For more information, see Palmer BF: 10 Nephrology: III Approach to the Patient with Renal Disease. ACP Medicine Online (www.acpmedicine.com). Dale DC, Federman DD, Eds. WebMD Inc., New York, April 2004

Management of Chronic Kidney Disease

13. A 57-year-old woman with hypertension, mitral valve prolapse with regurgitation, asthma, and a history of alcoholism presents to your office to establish primary care. Because the patient has hypertension, you order a basic metabolic profile and urinalysis as a part of your initial evaluation. The laboratory calls to notify you that the patient’s serum creatinine level is 2.3 mg/dl.

Which of the following statements regarding chronic kidney disease (CKD) is true?

A. CKD is defined as a glomerular filtration rate (GFR) of less than 30 ml/min/1.73 m2 for longer than 3 months
B. Persistently increased proteinuria in the setting of a normal or increased GFR signifies the presence of stage 1 CKD
C. Measurement of 24-hour creatinine clearance to assess GFR is more accurate than estimating GFR from the Modification of Diet in Renal Disease (MDRD) equation
D. Treatment of comorbid conditions, interventions to slow progression of kidney disease, and measures to reduce cardiovascular disease should begin during CKD stage 3

Key Concept/Objective: To understand the basic principles of the diagnosis and treatment of CKD

CKD is defined as either kidney damage or a GFR of less than 60 ml/min/1.73 m2 for longer than 3 months. The MDRD and Cockcroft-Gault equations provide useful estimates of GFR in adults. Clinical practice guidelines point out that clinicians should not use serum creatinine concentration as the sole means of assessing the level of kidney function. In addition, measurement of 24-hour creatinine clearance to assess GFR is not more accurate than estimating GFR from the MDRD equation. Evaluation of all patients with CKD should include testing for proteinuria. Persistently increased proteinuria is usually a marker of kidney damage; in the setting of a normal or increased GFR, it signifies the presence of stage 1 CKD. Treatment of comorbid conditions, interventions to slow progression of kidney disease, and measures to reduce cardiovascular disease should begin during stages 1 and 2. (Answer: B—Persistently increased proteinuria in the setting of a normal or increased GFR signifies the presence of stage 1 CKD)

14. A 46-year-old male patient with long-standing CKD, diabetes, and hypertension presents for routine follow-up. The patient is taking an ACE inhibitor and insulin. The patient’s GFR is 42 ml/min/1.73 m2, as calculated using the MDRD equation.

Which of the following statements regarding the management of complications associated with CKD
is false?

A. Potassium balance is generally maintained within normal limits until the GFR falls to less than 10 ml/min
B. When a patient is normotensive and maintains a constant weight with only trace edema, the patient has achieved the goal for salt intake
C. Initially, patients with CKD have an anion gap metabolic acidosis, but over time, they develop a non–anion gap metabolic acidosis
D. Initially in CKD, the parathyroid hormone level rises, resulting in fairly normal serum calcium and phosphate levels

Key Concept/Objective: To understand the basic principles of the management of the complications associated with CKD

The goal of salt intake should be a quantity that results in the patient’s being normotensive and maintaining a constant weight, with only trace edema present. Potassium balance is generally maintained within normal limits until the GFR falls to less than 10 ml/min. This balance is achieved by an increased potassium excretion rate per remaining nephron, as well as an increase in extrarenal potassium excretion, primarily effected via the colon. The development of hyperkalemia at higher levels of renal function suggests the presence of tubulointerstitial disease or disturbances in the reninangiotensin-aldosterone axis. As renal insufficiency progresses, patients typically become acidotic. Initially, the acidosis is of the non–anion gap type, but as renal insufficiency becomes far advanced, an anion gap acidosis supervenes. As GFR declines, the serum phosphate level begins to increase, causing a reciprocal decrease in the serum calcium concentration. In response, parathyroid hormone (PTH) is released, resulting in increased phosphate excretion in each of the remaining nephrons; thus, calcium and phosphorus levels return to normal. As renal function continues to decline, calcium and phosphorus levels remain within the normal range but at the expense of an everincreasing level of PTH. (Answer: C—Initially, patients with CKD have an anion gap metabolic acidosis, but over time, they develop a non–anion gap metabolic acidosis)

15. A 53-year-old man with type 2 diabetes presents to your office to establish primary care. You measure his serum creatinine level.

Which of the following statements is true regarding assessment of renal function?

A. A serum creatinine value that is within the normal range indicates that the patient’s GFR has declined by less than 25%
B. If the patient has renal insufficiency, use of creatinine clearance, as determined by 24-hour urine collection, to assess GFR can lead to an underestimation of the GFR
C. The Cockcroft-Gault formula estimates GFR while taking into account the increase in creatinine production with increasing weight and age
D. Averaging the urea and creatinine clearance values will provide a more accurate estimation of GFR than use of the creatinine clearance value alone
E. Creatinine clearance should provide an equally accurate estimation of GFR, regardless of the patient’s current creatinine level

Key Concept/Objective: To understand the various methods of estimating GFR

Various tools are available to assess renal function. The major limitation of the use of the serum creatinine level to assess GFR is that it cannot detect loss of renal function until the GFR has declined by more than 50%. Thus, a normal creatinine level does not rule out the possibility that the GFR has declined by more than 25%. The CockcroftGault formula takes into account the increase in creatinine production that occurs with increasing weight and the decrease in production that occurs with advancing age. The most accurate way of following renal function is to directly measure the GFR by measuring the clearance of a compound that is freely filtered by the glomerulus but that is neither secreted nor absorbed. Use of creatinine clearance provides a fairly accurate measure of GFR because at normal levels of renal function, only a small percentage of creatinine appears in the urine through tubular secretion. The bulk of creatinine is filtered by the glomerulus. With advancing renal insufficiency, however, the percentage of creatinine that reaches the final urine through tubular secretion increases. As a result, use of creatinine clearance tends to lead to progressively larger overestimations of GFR with advancing renal insufficiency. Urea is filtered by the glomerulus and is reabsorbed by the tubule; thus, use of urea clearance leads to underestimations of GFR. However, the extent to which urea clearance leads to underestimations of GFR is similar to the extent to which creatinine clearance leads to overestimations of GFR. Thus, taking the average of urea and creatinine clearance values will give a very accurate estimation of GFR. (Answer: D—Averaging the urea and creatinine clearance values will provide a more accurate estimation of GFR than use of the creatinine clearance value alone)

16. You are managing a man with long-standing hypertension, diabetes, and chronic renal insufficiency. He has gradually developed anemia and edema and has recently developed hyperkalemia and acidosis as the time approaches when he will require hemodialysis.

Which of the following statements is true regarding the etiology and management of these typical abnormalities associated with chronic renal insufficiency?
A. Alkali therapy can help treat the acidosis but is unlikely to improve the hyperkalemia
B. Constipation should be avoided because it can cause the hyperkalemia to worsen
C. The target hematocrit value for erythropoietin therapy is 30%
D. Alkali should not be administered as sodium bicarbonate to patients receiving aluminum-containing phosphate binders
E. Failure to respond to erythropoietin therapy is most commonly the result of underlying anemia of chronic renal disease

Key Concept/Objective: To understand the principles of management of metabolic and hematologic abnormalities in chronic renal failure

In chronic renal failure, hyperkalemia and acidosis are interrelated. Alkali therapy will certainly help to improve the acidosis and may improve the hyperkalemia through several mechanisms. First, alkalinization causes a shift of potassium into cells. Also, sodium bicarbonate enhances distal sodium delivery and therefore augments potassium secretion from the distal tubule. One should not administer citrate-containing alkali to patients receiving aluminum-containing phosphate binders, because citrate is known to enhance the gastrointestinal absorption of aluminum. In this setting, sodium bicarbonate diminishes the risk of aluminum toxicity. Constipation can cause hyperkalemia to worsen because potassium secretion by the colon is substantial in patients with advanced renal failure.
Severe anemia contributes to the development of left ventricular hypertrophy, which in turn is an important predictor of subsequent cardiac morbidity and mortality in patients receiving dialysis. Thus, early institution of erythropoietin therapy can improve dialysis outcomes. This is an important reason for timely referral of the patient with renal insufficiency to a nephrologist. A target hematocrit value of 30% in young patients who have no evidence of cardiovascular disease should provide relief of symptoms attributable to anemia. By contrast, older patients with comorbidities may benefit by targeting the hematocrit value closer to normal. Failure to respond to erythropoietin therapy is most commonly the result of iron deficiency. (Answer: B—Constipation should be avoided because it can cause the hyperkalemia to worsen)

17. A 34-year-old man with diabetes and hypertension comes for a check-up. His creatinine level is normal at 1.0 mg/dl, and he has microalbuminuria of 120 mg/24 hr.

Which of the following statements is true regarding the appropriate measures to slow progression of renal disease?
A. Aggressive control of hyperglycemia may be more likely to slow progression of renal disease in patients with type 1 diabetes mellitus than in patients with type 2 diabetes mellitus
B. The targeted blood pressure should be below 140/90 mm Hg
C. Because this patient has diabetes, microalbuminuria is predictive of progression of renal disease
D. Smoking is a risk factor for microalbuminuria because of its association with hypertension
E. Although not clearly of benefit, a low-protein diet can be prescribed with little concern about deleterious effects

Key Concept/Objective: To understand the risk factors for renal disease progression

Evidence clearly shows that aggressive control of hyperglycemia in patients with type
1 diabetes mellitus will reduce the occurrence of microalbuminuria and macroalbuminuria and will slow the progression of nephropathy. Control of hyperglycemia in patients with type 2 diabetes mellitus is more controversial, as there are conflicting results of this approach in the literature. This may be related to the fact that renal lesions resulting from type 2 diabetes are more heterogeneous than the typical lesion from type 1 diabetes. Because uncontrolled hypertension can contribute to the progression of renal disease, target blood pressure values have been established. These values vary slightly, depending on the source of the recommendation, but in general, a blood pressure of 130/80 mm Hg or less should be sought. Microalbuminuria is a risk factor for progression to end-stage renal disease in diabetic and nondiabetic patients with renal disease. Smoking is an independent risk factor for microalbuminuria in both hypertensive and normotensive patients. Finally, a low-protein diet can easily lead to malnutrition and calorie deficiency and therefore must be closely monitored. (Answer: A—Aggressive control of hyperglycemia may be more likely to slow progression of renal disease in patients with type 1 diabetes mellitus than in patients with type 2 diabetes mellitus)

For more information, see Palmer BF: 10 Nephrology: IV Management of Chronic Kidney Disease. ACP Medicine Online (www.acpmedicine.com). Dale DC, Federman DD, Eds. WebMD Inc., New York, September 2004

Glomerular Diseases

18. A previously healthy 54-year-old woman presents with a 3-week history of arthralgias and edema. Her examination is remarkable for a blood pressure of 170/106 mm Hg, bibasilar pulmonary crackles, and lower extremity edema. A freshly voided urine reveals red blood cells and red cell casts. A diagnosis of glomerulonephritis is made. Her serology is positive for antineutrophil cytoplasmic antibody (ANCA).

For this patient, a renal biopsy with immunofluorescent staining would be expected to show which of the following?
A. Positive staining for immune complexes
B. Positive staining for immune deposits IgG and C3
C. Positive staining for IgA and C3
D. Positive staining for linear deposition of IgG and C3
E. Negative staining for antibody or C3

Key Concept/Objective: To understand that ANCA–associated glomerulonephritis is not associated with staining for immunoglobulin, complement, or immune deposits

ANCA-associated glomerulonephritis involves a vasculitic process of the smalland medium-sized blood vessels that usually presents as a focal segmental necrotizing glomerulonephritis. Renal involvement is usually acute, severe, and progressive, and glomeruli contain crescents. ANCA-associated glomerulonephritis is one of the causes of rapidly progressive glomerulonephritis, which many authors consider a medical emergency. ANCA-associated glomerulonephritis can be limited to the kidney or coexist with systemic illness such as Wegener granulomatosis. In contrast to many other kinds of glomerulonephritis, immunofluorescent staining fails to reveal the presence of antibody, complement, or immune complexes. This type of glomerulonephritis is also referred to as pauci-immune glomerulonephritis. If the disease is left untreated, the prognosis is poor. Initial treatment consists of corticosteroids and immunosuppressive therapy. (Answer: E—Negative staining for antibody or C3)

19. A 45-year-old man presents for a routine examination. His history is remarkable for a bleeding peptic ulcer at age 30 that required transfusion of several units of packed red blood cells. He has been without peptic symptoms since then. He consumes no alcohol. His physical examination reveals a blood pressure of 154/98 mm Hg, confirmed on several occasions, but is otherwise not remarkable. His laboratory evaluation was remarkable for ALT and AST levels that were 2.5 times normal and a dipstick that was positive for blood and protein.

What should be the next step in this patient’s management?

A. Referral for a liver biopsy
B. Renal arteriogram
C. Helicobacter pylori antibody testing
D. Serum renin measurement
E. Serum antibody testing for hepatitis C

Key Concept/Objective: To understand the pathogenic link between chronic hepatitis C infection and glomerulonephritis

Membranoproliferative glomerulonephritis can present with either the nephrotic syndrome or the nephritic syndrome. Type 1 membranoproliferative glomerulonephritis is caused by immune complex deposition in the subendothelium, most commonly immune deposits from hepatitis C virus (HCV) antigens and cryoglobulins. Approximately one third of patients present with microscopic hematuria and nonnephrotic proteinuria, and another third presents with nephrotic-range proteinuria with a mild decrease in renal function. Hypertension is a very common finding on initial presentation. HCVassociated membranoproliferative glomerulonephritis is usually treated with antiviral therapy when remission is common and relapse is frequent. (Answer: E—Serum antibody testing for hepatitis C)

20. A 21-year-old woman of Peruvian descent presents with hypertension, fatigue, and microscopic hematuria. A renal biopsy demonstrates glomerulonephritis secondary to focal segmental glomerulosclerosis (FSGS).

Which of the following would be the most appropriate step to take next in the treatment of this patient’s disease?
A. Renal dialysis
B. Cyclosporine
C. Prednisone
D. Cyclophosphamide
E. Captopril

Key Concept/Objective: To understand that glucocorticoids represent the key initial medical therapy for patients diagnosed with FSGS

FSGS is one of the most common causes of nephrotic syndrome in adults. In addition to the signs and symptoms of nephrotic syndrome, hypertension and hematuria occur in 50% of adults with FSGS. As in all glomerular diseases, if a secondary cause can be found (e.g., HIV, heroin, analgesics), correcting the underlying cause is the first priority. However, many causes are idiopathic. The first line of therapy in adults with FSGS is prednisone. It should be emphasized that the duration of therapy is critical in preventing relapses. Shortening the duration of prednisone treatment from the recommended
6 to 8 months is a risk factor for relapse. Of patients treated initially with prednisone,
55% respond to this treatment. For the 45% who do not respond, cytotoxic agents (e.g., cyclophosphamide) should be tried. Interestingly, if a patient undergoes a renal transplantation, there is a 20% to 30% chance that native disease will recur in the transplanted kidney within 3 years. (Answer: C—Prednisone)

21. A 45-year-old man presents with acute onset of flank pain and hematuria. He gives a history of several months of increasing peripheral edema in his lower extremities. His urinary protein-to-creatinine ratio is 3.9.

What would be the best step to take next in this patient’s management?
A. Renal ultrasound and duplex scan
B. Gram-negative antimicrobial therapy
C. Intravenous furosemide
D. Rapid-sequence intravenous pyelogram
E. ASO titer

Key Concept/Objective: To understand that patients with nephrotic syndrome are at risk for thrombosis of the renal vein

It is estimated that 10% to 40% of patients with nephrotic syndrome will develop arterial or venous thromboembolism. Urinary losses of antithrombin III are thought to contribute to the pathogenesis of this complication. Renal vein thrombosis is most commonly found in membranous nephropathy, where it may occur in up to 50% of patients. Some authors recommend the use of prophylactic low-dose warfarin when the plasma albumin concentration is less than 2 g/dl. The diagnosis of renal vein thrombosis is best made via Doppler ultrasonography. The urinary protein-to-creatinine ratio of 3.9 is consistent with 3.9 g of protein a day and is in the nephrotic range. (Answer: A— Renal ultrasound and duplex scan)

22. A 36-year-old hypertensive man develops macroscopic hematuria 24 hours after the onset of pharyngitis. The patient’s brother had a history of poststreptococcal glomerulonephritis at age 6 after a streptococcal infection of the throat.

What is the most likely explanation for this patient’s hematuria?
A. Poststreptococcal glomerulonephritis
B. Glomerulosclerosis
C. IgA nephropathy
D. Henoch-Schonlein purpura (HSP)
E. Renal vein thrombosis

Key Concept/Objective: To understand the relation between hematuria and mucosal infections in patients with IgA nephropathy

Patients with IgA nephropathy typically present with nephritic-like symptoms that derive from deposition of IgA in the glomeruli. It is the leading cause of glomerulonephritis worldwide. The male-to-female ratio is 3:1. The classic presentation in up to 50% of patients with IgA nephropathy is episodic macroscopic hematuria within 24 hours of a mucosal infection of the upper respiratory tract. The majority of the rest of patients with IgA nephropathy present with persistent asymptomatic microscopic hematuria. This differs from the hematuria of poststreptococcal glomerulonephritis, which is delayed by 2 to 3 weeks following pharyngitis. The macroscopic hematuria usually resolves within days. (Answer: C—IgA nephropathy)

For more information, see Shankland SJ: 10 Nephrology: V Glomerular Diseases. ACP Medicine Online (www.acpmedicine.com). Dale DC, Federman DD, Eds. WebMD Inc., New York, September 2004

Acute Renal Failure

23. A 25-year-old woman presents to your clinic with fatigue of 1 week’s duration. She thinks there was blood in her urine on two occasions after excessive exercise. Physical examination is unremarkable except for some mild muscle tenderness. Urinalysis is positive for 3+ blood. The blood urea nitrogen (BUN) level is 18 mg/dl, and the creatinine level is 1.1 mg/dl.

What is the most likely cause of this patient’s symptoms?
A. Postinfectious glomerulonephritis
B. Myoglobinuria caused by rhabdomyolysis
C. IgA nephropathy
D. Wegener granulomatosis

Key Concept/Objective: To know the signs and symptoms of rhabdomyolysis

Over the past 50 years, our understanding of rhabdomyolysis has significantly broadened. The most common causes are trauma or other disorders that lead to muscle injury; excessive muscle activity, as occurs during seizures or strenuous exercise; use of medications; and electrolyte disorders. Recent increased use of HMG-CoA (3-hydroxy-3methylglutaryl coenzyme A) reductase inhibitors (statins) has been associated with greater incidence of rhabdomyolysis. Diagnosis is made by symptoms of muscle pain, dark-brown urine without red cells on urinalysis, and elevated creatine kinase levels. Approximately 30% of patients develop acute renal failure (ARF) and other electrolyte imbalances; in these patients, early diagnosis and treatment are the keys to minimizing ARF. In patients who are acutely ill, volume repletion and close monitoring of urine output are imperative. Although, in the past, alkalinization was a mainstay in the treatment of rhabdomyolysis, it is no longer considered mandatory; in some studies, use of urinary alkalinization was not found to be superior to use of saline. (Answer: B— Myoglobinuria caused by rhabdomyolysis)

24. A 53-year-old woman presented to the emergency department with a cough, fever, and yellow sputum production; she had been experiencing these symptoms for 1 week. On physical examination, crackles were heard in the left lower and middle lung zones, and the patient experienced pain on inspiration. Laboratory results were as follows: Na, 128 mEq/L; K, 2.8 mEq/L; BUN, 25 mg/dl; creatinine, 1.1 mg/dl. A chest radiograph showed a consolidation in the left lower lobe. The patient was admitted and treated with I.V. antibiotics. On the day after admission, her blood pressure dropped to 90/75 mm Hg, and she became confused. After I.V. administration of normal saline, the blood pressure increased to 105/70 mm Hg and there was improvement in the patient’s mental status. A repeat chest radiograph showed continued consolidation in the left lower lobe and progression in the right middle lobe. The patient’s temperature was 103.2° F (39.5° C). Gentamicin was added. Six hours later, the patient’s creatinine level is 2.1 mg/dl, and her BUN is 32 mg/dl.

What is the most likely cause of the increase in this patient’s creatinine level?
A. IgA nephropathy
B. Gentamicin nephrotoxicity
C. Acute tubular necrosis (ATN) caused by hypotension
D. Syndrome of inappropriate antidiuretic hormone (SIADH) associated with pneumonia

Key Concept/Objective: To be able to differentiate aminoglycoside toxicity from hypotensioninduced ATN

In the hospitalized patient, ATN is the most common cause of ARF. However, in most ARF patients, multiple insults complicate the clinical picture. Common contributing factors include sepsis and nephrotoxins in addition to the usual prerenal and postrenal azotemia. Determining the actual cause among various possible causes of ARF is often difficult. In this patient, sepsis, hypotension, and nephrotoxins may have contributed to the ARF. ATN from hypotension is the most likely cause, given the time and rapidity of onset. If her BUN and creatinine levels had risen a week later, aminoglycoside toxicity would have been a likely cause; however, 6 hours is too short an interval for this patient’s ARF to have been caused by aminoglycoside toxicity. IgA nephropathy can occur rapidly; however, it is more likely to occur over a period of weeks than hours. Severe SIADH could potentially complicate the clinical picture over time. (Answer: C— Acute tubular necrosis [ATN] caused by hypotension)

25. A 24-year-old African-American woman comes to your office with complaints of fatigue, wrist and knee pain, and progressive lower extremity edema. She first noticed these symptoms 2 months ago; before then, she had been in good health. On physical examination, her blood pressure is elevated to 185/105 mm Hg, she has no fever, and her heart rate and respiratory rate are normal. Results of cardiovascular, lung, and abdominal examinations are normal. All her joints are normal, and no redness, warmth, or effusions are noted. You corroborate pitting edema up to her midshin bilaterally. Laboratory results are as follows: creatinine, 3.5 mg/dl; BUN, 56 mg/dl. The levels of all the electrolytes are normal.

Which of the following results of urinalysis would be most consistent with this patient’s clinical picture?

A. Moderate number of hyaline and finely granular casts
B. Presence of moderate to severe proteinuria (3+ to 4+), red blood cells (RBCs), and RBC casts
C. Dipstick is positive for blood with few or no RBCs
D. Dirty-brown granular casts and granular epithelial cells, both free and in casts
E. Relatively normal results with no cells or few cells and no casts

Key Concept/Objective: To understand the value of microscopic examination of the urine in determining the etiology of ARF

Urinalysis can provide invaluable information for patients with ARF. Prompt processing of the specimen is of paramount importance. Patients with myoglobinuria or hemoglobinuria characteristically have positive findings on dipstick testing for blood and an absence of RBCs on microscopic examination of the urine. For patients with postrenal azotemia and those with hepatorenal syndrome, findings on urinalysis are relatively benign, and there is an absence of casts and cells. Patients with prerenal azotemia demonstrate hyaline and finely granular casts unless their condition has progressed to ATN. In patients with ATN, regardless of the etiology (renal or prerenal), the urine sediment has characteristic dirty-brown granular casts and both free renal epithelial cells and epithelial cell casts. This patient is likely to have proliferative glomerulonephritis; the urine sediment of such patients exhibits significant proteinuria, RBCs, and RBC casts. The differential diagnosis for proliferative glomerulonephritis includes connective tissue diseases, systemic vasculitis, postinfectious glomerulonephritis, and other diseases. (Answer: B—Presence of moderate to severe proteinuria [3+ to 4+], red blood cells [RBCs], and RBC casts)

For more information, see Shaver MJ, Shah SV: 10 Nephrology: VI Acute Renal Failure. ACP Medicine Online (www.acpmedicine.com). Dale DC, Federman DD, Eds. WebMD Inc., New York, June 2002

Vascular Diseases of the Kidney

26. A 32-year-old woman presents to you after a recent hospital admission for flash pulmonary edema. She was diagnosed with hypertension several months ago. Her blood pressure remains poorly controlled despite compliance with a regimen of hydrochlorothiazide, amlodipine, and metoprolol. She denies having headache and palpitation. Her physical examination is remarkable for a blood pressure of 204/106 mm Hg in the left arm and bilateral abdominal bruits. You consider the diagnosis of renal artery stenosis (RAS) secondary to fibromuscular dysplasia (FMD).

Which of the following statements regarding RAS and FMD is true?

A. Renal ultrasonography should be the first step in the evaluation of RAS because a finding of symmetrical kidneys precludes the need for further testing
B. Angioplasty with stenting has become the most common method of managing FMD associated with hypertension and renal insufficiency; this procedure completely cures more then 50% of patients with hypertension and improves renal function in over one third
C. The segmental nature of medial fibroplasia, the most common subtype of FMD, results in the classic so-called beads-on-a-string appearance in the proximal third of the main renal artery
D. Surgical repair of aneurysms is required if their diameter is greater than 1.5 cm or if the patient has uncontrolled hypertension or is pregnant

Key Concept/Objective: To understand the diagnosis and treatment of FMD

Medial fibroplasia, the most common subtype of FMD, is characterized by a predominance of fibrotic material in the media, with sparing of the intima and adventitia. It affects the distal two thirds of the main renal artery and its branches. In patients with a compatible clinical picture, evaluation for RAS starts with renal ultrasonography to measure kidney size. If there is a large discrepancy in kidney sizes (i.e., > 2 cm in the longitudinal axis), significant arterial stenosis is or was likely. Even if the ultrasound scan shows that the kidneys are equal in size, further diagnostic testing is required. The choice of procedures is determined by the level of renal function: patients with a serum creatinine level below 2 mg/dl should undergo renography; those with a serum creatinine above 2 mg/dl should undergo magnetic resonance angiography (MRA). The gold standard for the diagnosis of RAS remains a renal arteriogram. Percutaneous intervention has been the standard of care, but large comparative trials are not feasible, given the relative rarity of these conditions. Angioplasty and stenting completely cure hypertension in about 22% of patients. Surgical repair of aneurysms (the “beads” seen on arteriography) is required if their diameter is greater than 1.5 cm; it is also required if the patient has uncontrolled hypertension or is pregnant. (Answer: D—Surgical repair of aneurysms is required if their diameter is greater than 1.5 cm or if the patient has uncontrolled hypertension or is pregnant)

27. A 58-year-old man known to have nephrotic syndrome presents to the emergency department. For several days, he has been experiencing low back pain and for the past several hours, he has been experiencing hematuria and shortness of breath. The patient is tachypneic, with an oxygen saturation of 92% on 4 L of oxygen via nasal cannula. A CT angiogram reveals pulmonary thromboembolism.

For this patient, which of the following statements regarding renal vein thrombosis (RVT) is true?

A. RVT is most frequently associated with idiopathic and secondary membranous nephropathy; of these patients, 30% may have RVT
B. In addition to acute lower back pain and hematuria, most patients present with some degree of renal insufficiency
C. Doppler ultrasonography is the most common modality used in the diagnosis of RVT
D. For patients with RVT, a 6-month course of warfarin is indicated

Key Concept/Objective: To understand the prevalence, clinical presentation, diagnostic modalities, and treatment of RVT

RVT has been most frequently associated with idiopathic and secondary membranous nephropathy; 30% of these patients may have RVT. Pulmonary embolism may develop in up to 30% of patients with RVT, although alarmingly, the vast majority of these patients are asymptomatic. The classic clinical presentation of RVT is acute lower back pain and gross hematuria. Patients typically do not have renal insufficiency or hypertension. RVT can be diagnosed by computed tomography, magnetic resonance imaging, and contrast venography. Doppler ultrasound imaging is notoriously operator dependent and therefore should not be used for the diagnosis of RVT. Anticoagulation with warfarin is indicated for patients with RVT. The therapeutic goal is an INR of 2 to 3. The appropriate duration of therapy is likely lifelong. (Answer: A—RVT is most frequently associated with idiopathic and secondary membranous nephropathy; of these patients, 30% may have RVT)

28. A 48-year-old white man with no significant medical history presents to your office with fever, weight loss, malaise, and arthralgia. He has had these symptoms for several months. Over the past few weeks, he has developed a purplish rash over his lower extremities and several sores on his toes. He is afebrile, but his blood pressure is 187/92 mm Hg and his heart rate is 97 beats/min. Livedo reticularis and digital ischemia are noted on examination. Laboratory results are significant for a potassium level of 3.2 mEq/L, a blood urea nitrogen (BUN) value of 28 mg/dl, and a creatinine level of 2.3 mg/dl.

Which of the following statements regarding polyarteritis nodosa (PAN) is true?
A. Serologic tests for PAN are diagnostic; most patients exhibit a positive enzyme-linked immunosorbent assay (ELISA) titer to antibodies against serine protease 3 and myeloperoxidase
B. The pathogenesis of PAN is unclear, although there appears to be an association with hepatitis C infection
C. ACE inhibitors and angiotensin receptor blockers (ARBs) should be used cautiously in patients with PAN because renal involvement may produce a functional equivalent of RAS
D. In approximately 90% of patients with PAN, remission is achieved with high-dose steroids

Key Concept/Objective: To understand the pathogenesis, diagnostic criteria, and treatment of PAN

The pathogenesis of PAN is unclear. There appears to be an association with hepatitis B viral infection. The diagnosis of PAN is made by demonstration of the characteristic lesion in an artery. Serologic tests are not diagnostic in PAN, but low-titer antibodies to rheumatoid factor and nuclear antigen may be present. Immunofluorescence antibody staining for cytoplasmic and perinuclear antineutrophil cytoplasmic autoantibodies (ANCAs) may be positive, but the more specific test—serum ELISA titers for antibodies against both serine protease 3 (PR3) and myeloperoxidase—is negative. If left untreated, patients with PAN have a poor prognosis. In such cases, patients are at risk for ischemia of numerous organ systems; the major causes of morbidity and mortality include renal failure, mesenteric ischemia, and cerebrovascular disease. Corticosteroids and cytotoxic agents have been the mainstays of therapy for idiopathic PAN, although the optimal therapy remains unknown. Approximately 50% of patients with idiopathic PAN achieve remission with high-dose steroids (e.g., prednisone, 1 mg/kg/day) for 3 to 6 months. Cyclophosphamide, either intravenous (0.6 g/m2) or oral (2 mg/kg/day), for up to 1 year is used in patients whose disease does not respond to steroids or in patients who are at risk for serious complications. ACE inhibitors and ARBs should be used cautiously in patients with PAN, because renal involvement may produce a functional equivalent of classic renal artery stenosis. (Answer: C—ACE inhibitors and angiotensin receptor blockers [ARBs] should be used cautiously in patients with PAN because renal involvement may produce a functional equivalent of RAS)

29. A 54-year-old man presents with a 4-day history of low-grade fever and confusion. He was previously healthy. His physical examination is significant for pallor and ecchymoses. Laboratory studies reveal a hemoglobin of 7.6 g/dl, a WBC of 8,200/µl, and a platelet count of 12,000/µl. The peripheral blood smear shows schistocytes and a decreased number of platelets.

For this patient, which of the following statements regarding thrombotic microangiopathies (TMAs) is true?

A. When plasma activity of metalloprotease (ADAMTS-13) is elevated, von Willebrand antigens predominate; those antigens bind to platelets and cause aggregation and thrombi in the small vessels
B. A presumptive diagnosis of thrombotic thrombocytopenic purpura (TTP) is often based on the presence of thrombocytopenia, schistocytes, and prolonged prothrombin time (PT) and partial thromboplastin time (PTT)
C. Hemolytic-uremic syndrome (HUS) is characterized by platelet aggregation and the presence of large von Willebrand multimers
D. The clinical presentation of antiphospholipid syndrome (APS) generally comprises a single thrombotic event in the arterial system

Key Concept/Objective: To understand the pathogenesis and clinical presentations of the various thrombotic microangiopathies

The classic TMAs include TTP and HUS. The critical role of ADAMTS-13 (a disintegrin and metalloprotease with thrombospondin type 1 motif) in the pathogenesis of TTP has emerged in the past 10 years. ADAMTS-13, which is found on the surface of endothelial cells, normally cleaves large multimers of the von Willebrand antigen as they are secreted by the cell. These large multimers bind more efficiently than the cleaved von Willebrand antigen to platelets (at the Iba component of platelet glycoprotein Ib/IX/V vWA receptor). Plasma ADAMTS activity must be severely depressed for TTP to develop. The level of ADAMTS-13 plasma activity is modestly depressed in patients with liver disease, disseminated cancer, chronic metabolic and inflammatory conditions, and pregnancy, as well as in newborns. When plasma activity of ADAMTS-13 falls to less than 5% of normal, these large von Willebrand antigens predominate, bind to platelets, and cause aggregation and thrombi in small vessels. Although TTP is often defined by the classic pentad of findings, a presumptive diagnosis can often be based on a triad of laboratory observations: thrombocytopenia, schistocytes, and elevated serum lactate dehydrogenase (from shredded erythrocytes). Conditions other than TMA may also be difficult to distinguish from TTP. HUS is defined as a TMA that occurs after exposure to Escherichia coli 0157:H7 or other enteropathogenic serotypes of E. coli, Shigella dysenteriae, and, occasionally, some other enteropathogenic organism. Systemically, Shiga toxins produced by the bacteria play two key roles in the development of thrombi composed of platelets and von Willebrand antigen. First, they impair the secretion of ADAMTS-13 through an unspecified mechanism, resulting in large von Willebrand multimers. Second, they activate platelet adherence via the glycoprotein Iba component of glycoprotein Ib/Ix/V1 and V2, which may then promote von Willebrand antigen and platelet aggregation. The clinical presentation of APS most often comprises a single thrombotic event in either the arterial or the venous system. Deep vein thrombosis of the lower extremities is the most common occurrence. (Answer: C—Hemolytic-uremic syndrome [HUS] is characterized by platelet aggregation and the presence of large von Willebrand multimers)

30. A 49-year-old man reports coughing up 2 to 4 oz of blood several times in the past few weeks. He has a history of chronic nasal congestion and sinus infections. On review of systems, he reports worsening fatigue, mild fevers to 100° F (37.8° C), cough, dyspnea, and a 10-lb weight loss over the past 3 to 4 months. He works as a manager for a grocery store chain, has never used I.V. drugs, and has been monogamous with his wife over the 10 years they have been married. His blood pressure is 130/80 mm Hg. A chest examination reveals scattered rales, and his skin is without rash. Laboratory test results are as follows: HCT, 33; WBC, 12, with normal differential; ESR, 98; creatinine, 2; BUN, 30; antineutrophilic cytoplasmic antibodies are present in a cytoplasmic staining distribution. Urinalysis shows moderate hemoglobin and protein levels, microscopic exam of the urine shows red cell casts, and sputum cultures are negative for acid-fast bacilli and bacteria. Thoracoscopic lung biopsy reveals necrotizing granulomas.

What is the most likely diagnosis for this patient?
A. Sarcoidosis
B. Tuberculosis
C. Polyarteritis nodosa
D. Goodpasture syndrome
E. Wegener granulomatosis

Key Concept/Objective: To know the characteristic presentation of Wegener granulomatosis and the specificity of a positive cANCA test result for this disorder

Wegener granulomatosis classically occurs in middle-aged adults as a pulmonary renal syndrome with hemoptysis, pulmonary infiltrates, and glomerulonephritis with red cell casts. In addition, antineutrophilic antibodies are present in over 90% of patients with Wegener granulomatosis and are a relatively specific indication of Wegener granulomatosis when present in a cytoplasmic staining distribution. Lung biopsy that shows necrotizing granulomas is diagnostic. Sarcoidosis and TB cause noncaseating granulomas, not necrotizing granulomas in lung tissue. Sarcoidosis usually causes pulmonary fibrosis, bronchiectasis, and cavitation, along with mediastinal adenopathy. Renal sarcoidosis usually causes renal insufficiency through hypercalcemia or tubular dysfunction from granulomatous interstitial nephritis. Although pulmonary TB may cause hemoptysis, it more characteristically causes a focal infiltrate in the upper lobe of the lung or appears in a miliary pattern on chest x-ray. Furthermore, TB involving the kidneys more likely causes significant pyuria. Classic polyarteritis does not involve the lungs and is characterized by a perinuclear, not cytoplasmic, ANCA staining pattern. Goodpasture syndrome is a pulmonary renal syndrome with a presentation similar to that of Wegener granulomatosis, but in Goodpasture syndrome, ANCA test results are negative. (Answer: E—Wegener granulomatosis)

31. What treatment would be most appropriate for the patient described in Question 30?
A. Prednisone
B. Azathioprine
C. Four-drug therapy for TB
D. Trimethoprim-sulfamethoxazole
E. Cyclophosphamide plus prednisone

Key Concept/Objective: To know that the appropriate treatment of Wegener granulomatosis is cyclophosphamide in combination with prednisone

Early treatment with the combination of cyclophosphamide and prednisone is the most effective way to prevent rapid progression to renal failure in patients with Wegener granulomatosis. This combination can also induce remission in up to 75% of patients. Prednisone may cause temporary clinical improvement but rarely results in remission. Neither azathiaprine nor four-drug TB therapy would be useful against Wegener granulomatosis. In patients with Wegener granulomatosis who are in remission, trimethoprim-sulfamethoxazole is used to prevent relapse of disease; it is not used in patients with active disease. (Answer: E—Cyclophosphamide plus prednisone)

32. A 44-year-old woman reports severe right calf pain, which has been worsening over the past week. She occasionally takes acetaminophen and occasionally uses alcohol but does not use cigarettes or I.V. drugs; she has had a monogamous relationship with a female partner for 8 years. She has been feeling under the weather for several months, with fatigue, unintentional weight loss of 8 lb, and postprandial abdominal discomfort. She denies having cough, dyspnea, hemoptysis, chest pain, change in bowel habits, urinary symptoms, or rash. On examination, the patient’s temperature is 99.8° F (37.7° C), her blood pressure is 160/100 mm Hg, and there are abrasions on her right knee and palm from a recent fall. On neurologic examination, the patient has marked weakness of right foot dorsiflexion. Skin examination reveals livedo reticularis over the patient’s back and lower extremities. Urinalysis results are normal, ESR is 87, creatinine is 1.9, rheumatoid factor and antinuclear antibody test results are negative, and chest xray is normal.

What is the most likely diagnosis for this patient?
A. Microscopic polyarteritis
B. Polyarteritis nodosa
C. Sarcoidosis
D. Cholesterol embolism
E. Systemic lupus erythematosus

Key Concept/Objective: To know the presentation of polyarteritis nodosa

Both polyarteritis nodosa and microscopic polyarteritis can cause neurologic deficits, livedo, renal compromise, and systemic symptoms of fatigue, fever, and weight loss. However, because polyarteritis nodosa affects larger vessels, it can cause downstream glomerular ischemia, thereby activating the renin-angiotensin system and raising blood pressure without causing an active urine sediment. Microscopic polyarteritis, on the other hand, affects smaller vessels, causing glomerular necrosis and the resulting active urine sediment of red cell casts and protein, without raising blood pressure. Although sarcoidosis can cause nerve and renal injury as well as systemic symptoms, sarcoidosis is unlikely to be the cause of this patient’s symptoms because 90% of patients with sarcoidosis have pulmonary involvement. Cholesterol emboli can cause livedo and pain in the legs or abdomen, although it should not cause a footdrop. It is usually seen in patients with significant atherosclerotic disease or risk factors for atherosclerosis who have recently undergone an invasive angiographic procedure. Lupus could explain the systemic symptoms, the neurologic deficit, and livedo, although it is unlikely with a negative antinuclear antibody test result. (Answer: B—Polyarteritis nodosa)

33. For which of the following tests would a positive result be diagnostic for the condition of the patient in Question 32?
A. Angiography
B. Renal biopsy
C. ANCA
D. Electromyography
E. Abdominal CT scan

Key Concept/Objective: To know that renal or celiac angiographic findings can be diagnostic of polyarteritis nodosa when microaneurysms are present

Celiac or renal angiographic findings of microaneurysms and irregular, segmental constriction of the larger vessels with tapering and occlusion of smaller intrarenal arteries are diagnostic of classic polyarteritis nodosa. In the absence of active urine sediment, renal biopsy is unlikely to be diagnostic. In addition, because the findings associated with the vasculitides often overlap, renal biopsy findings are not usually diagnostic. Abdominal CT scanning is not sensitive enough to pick up the microaneurysms of polyarteritis nodosa. ANCA with a perinuclear staining pattern is more likely to be present in microscopic polyarteritis than in the classic form of polyarteritis nodosa. Electromyopathy can assist in determining whether nerve damage is axonal or demyelinating, although it is rarely diagnostic. (Answer: A—Angiography)

34. A 21-year-old college student reports abdominal pain, bilateral ankle and knee pain, bloody urine, and a worsening rash that began on his lower legs and has spread to his trunk. He denies having had any recent infectious exposures or infections; he also denies using I.V. drugs and having any sexual contacts. On examination, the patient is afebrile, his blood pressure is 120/80 mm Hg, and his pulse is 76 beats/min. Skin examination reveals raised, indurated, purple coalescing papules on his anterior shins, lower legs, and abdomen. Urinalysis shows moderate levels of hemoglobin and protein with red blood cell casts on microscopic examination. Stool guaiac results are positive; CBC is normal, with a normal WBC differential; creatinine is 0.8; and the results of testing for antinuclear antibodies, cryoglobulins, rheumatoid factor, and hepatitis antibodies are negative. Skin biopsy results reveal an intense neutrophilic infiltrate surrounding dermal blood vessels, confirming leukocytoclastic vasculitis.

Which of the following is most likely to be true about this man’s condition?
A. Renal biopsy is diagnostic for Henoch-Schonlein purpura
B. Polyclonal IgG deposits on skin biopsy confirm Henoch-Schonlein purpura
C. Empirical treatment for gonococcal infection should be started
D. The extent of renal involvement is the most important prognostic factor
E. Prednisone and cyclophosphamide therapy should be started as soon as possible

Key Concept/Objective: To know the diagnosis and prognosis of Henoch-Schonlein purpura

Henoch-Schonlein purpura is diagnosed on the basis of the classic tetrad of skin rash, abdominal pain, arthralgias and arthritis, and glomerulonephritis. The extent of renal involvement is the most important prognostic factor in Henoch-Schonlein purpura. Renal biopsy results are not diagnostic of Henoch-Schonlein purpura, as such results can be identical with the results obtained in cases of IgA nephropathy with IgA deposition in the mesangium and in cases involving severe crescent formation. Skin biopsy results also show IgA (not IgG) deposition on immunofluorescence. This patient does not have any risk factors or signs of sepsis; if there is any suspicion that gonococcal or rickettsial infection is causing the palpable purpura, empirical therapy should be started immediately. Most cases of Henoch-Schonlein purpura resolve spontaneously, although prednisone and cyclophosphamide should be considered for use in the few patients with acute renal failure. (Answer: D—The extent of renal involvement is the most important prognostic factor)

For more information, see Kshirsager A, Falk RJ: 10 Nephrology: VII Vascular Diseases of the Kidney. ACP Medicine Online (www.acpmedicine.com). Dale DC, Federman DD, Eds. WebMD Inc., New York, December 2004

Tubulointerstitial Diseases

35. A 67-year-old black man with a history of tobacco abuse and ethanol abuse is admitted for gradually worsening esophageal dysphagia complicated by a 1-day history of shortness of breath, productive cough, and fever. On examination, the patient has a temperature of 101.5° F (38.6° C); he is tachypneic and has signs of consolidation in his right posterior lung field. Chest radiography reveals a right lower lobe infiltrate consistent with aspiration pneumonia. He is placed on piperacillin-tazobactam and oxygen, and he gradually improves. By hospital day 3, he experiences defervescence, but on hospital day 10 he is noted to again have a fever (100.8° F [38.2° C]). In addition, the patient has a rash, and peripheral blood eosinophilia and acute renal insufficiency are present.

Which of the following statements concerning this patient’s condition is most correct?
A. This patient will likely progress to end-stage renal disease
B. Standard of care would include stopping the piperacillin-tazobactam and starting high-dose I.V. solumedrol
C. Another ß-lactam antibiotic can be safely substituted for piperacillin
D. Urinalysis will most likely reveal sterile pyuria, mild proteinuria, and hematuria
E. Most patients with this disorder become oligoanuric

Key Concept/Objective: To understand the clinical manifestations and management of acute interstitial nephritis (AIN)

Virtually all ß-lactam antibiotics (i.e., penicillins and cephalosporins) can produce AIN. It usually occurs after several weeks of high-dose antibiotic therapy. Classically, patients exhibit a triad of hypersensitivity reactions: rash, fever, and eosinophilia. The secondary fever associated with AIN usually occurs after defervescence from the original infectious disease and during the onset of the allergic reaction. Urinary findings in patients with AIN include the nonspecific findings of sterile pyuria and mild proteinuria, as well as the more significant finding of hematuria, which in some patients may be gross. Eosinophils may be found in the urine sediment on Wright or Hansel staining in over 75% of cases. The pathogenesis of ß-lactam-associated AIN remains unknown. The disease is not dose related and occurs in only a small number of the millions of people taking ß-lactam drugs each year. It can recur or be exacerbated on rechallenge with a second ß-lactam drug. ß-Lactam-associated AIN is treated by discontinuing the drug and avoiding other ß-lactam antibiotics. Most patients regain renal function, and many regain baseline renal function. Only a minority of patients with AIN are oliguric. The use of corticosteroids to treat renal failure associated with AIN remains controversial. No randomized, controlled trials have yet proved that corticosteroid therapy has any advantages over discontinuance of medication. (Answer: D—Urinalysis will most likely reveal sterile pyuria, mild proteinuria, and hematuria)

36. A 65-year-old man with hypertension and reflux disease presents to your office for routine follow-up. He has no complaints. Laboratory data reveal an increased serum potassium level of 5.8 mEq/L. On questioning, you learn that the patient has a history of hesitancy, dribbling, and a decrease in the urinary stream.

Which of the following statements pertaining to renal interstitial damage from physical factors is false?
A. For patients with vesicoureteral reflux, medical therapy is unhelpful, and surgical intervention should be recommended immediately
B. In patients older than 60 years, the most common causes of obstructive uropathy are benign prostatic hypertrophy and prostatic or gynecologic malignancies
C. Ultrasonography is the diagnostic test of choice for obstructive nephropathy
D. Before diagnosing a patient as having vesicoureteral reflux disease, any urinary tract infection should be treated if present, and a repeat voiding cystourethrogram should be performed

Key Concept/Objective: To know that vesicoureteral reflux can cause chronic tubulointerstitial damage, known as reflux nephropathy

Vesicoureteral reflux can cause chronic tubulointerstitial damage, known as reflux nephropathy. When infection is present, even low-pressure refluxed urine that reaches the kidney can produce chronic interstitial inflammation and scarring. Presenting features can include such signs of urinary tract infection as back or flank pain, fever, and dysuria. Hypertension, when present, is often associated with high levels of renin, which may derive from the segmental areas of scarred parenchyma. Patients often have a urinary concentrating defect, leading to nocturia and polyuria. Reflux is demonstrated with a voiding cystourethrogram. Because urinary tract infection may be associated with reflux, it is best to wait several weeks after treating a urinary tract infection before trying to diagnose reflux nephropathy. Treatment of low-grade reflux is medical: longterm antibiotic therapy is used to sterilize the urine and prevent reinfection. Many persons with mild reflux undergo spontaneous remission with time. More severe reflux may require surgical intervention, although most comparative studies have not found an advantage for surgical intervention over medical therapy. Obstructive nephropathy may produce chronic interstitial damage, especially when the obstruction is partial or intermittent and longstanding. In persons older than 60 years, benign prostatic hypertrophy and prostatic and gynecologic cancers are common etiologies. The pathologic findings show dilatation of the collecting ducts and distal tubules. Ultrasonography usually shows dilatation of the urinary collecting system and hydronephrosis. Treatment consists of relieving the obstruction. (Answer: A—For patients with vesicoureteral reflux, medical therapy is unhelpful, and surgical intervention should be recommended immediately)

37. A 38-year-old man comes to your office for evaluation of a urinalysis that revealed proteinuria. Further evaluation demonstrated proteinuria in the nonnephrotic range and a creatinine level of 1.8 mg/dl. The patient has celiac disease with steatorrhea, which was diagnosed many years ago. You suspect he has chronic interstitial nephritis that is associated with celiac disease.

Which of the following scenarios is NOT associated with tubulointerstitial nephritis?
A. A patient who several years ago underwent stomach bypass surgery for morbid obesity
B. A 35-year-old woman who has non-Hodgkin lymphoma with bulky disease and is 2 days post chemotherapy
C. A patient with vitamin D deficiency who presents with tetany and paresthesias
D. A 68-year-old man with hypertension who ingested moonshine for 40 years

Key Concept/Objective: To understand the metabolic disturbances that can produce renal tubulointerstitial abnormalities, as well as environmental factors that can cause renal damage

Oxalic acid is a dicarboxylic end product of metabolism that is removed from the body only by renal excretion. Precipitation of calcium oxalate can produce nephrolithiasis, acute renal failure, or chronic tubulointerstitial damage. Patients with steatorrhea from various intestinal diseases—including celiac disease, Crohn disease, Wilson disease, and chronic pancreatitis—or from small bowel resection or bypass operations for obesity may hyperabsorb oxalate from the large bowel. The pathogenesis of oxalate hyperabsorption involves the abnormal binding of intraluminal gut calcium to fats, which frees more oxalate for absorption. In addition, the solubilizing effect of bile acids on the large bowel permits greater absorption of oxalate. The result can be nephrolithiasis, acute renal insufficiency, or chronic tubulointerstitial damage. Hyperuricemia and hyperuricosuria can lead to uric acid and acute oliguric renal failure from urate deposition. These conditions occur after massive release of uric acid, as occurs in the tumor lysis syndrome or chronic renal failure. Hypercalcemia and hypercalciuria can also produce a number of adverse renal effects. Hypercalcemia may lead to chronic tubulointerstitial damage. Chronic elevations of the calcium level can lead to calcium salt deposition in the tubules and interstitial regions; such depositions are associated with chronic interstitial inflammation, tubular atrophy, and fibrosis. Two heavy metals—lead and cadmium—clearly produce tubulointerstitial damage. Exposure to lead can occur from the ingestion of lead-based paints, the ingestion of beverages stored in crystal decanters made with lead, the ingestion of moonshine whiskey made in a lead-containing still, the manufacture or destruction of lead batteries, or the ingestion of lead-containing aerosols in the workplace. (Answer: C—A patient with vitamin D deficiency who presents with tetany and paresthesias)

38. A 56-year-old black man presents with bone pain, anemia, hypercalcemia, and renal insufficiency. Bone marrow biopsy indicates a diagnosis of multiple myeloma.

Which of the following mechanisms does NOT classically cause renal damage in patients with multiple myeloma?
A. Excessive filtration of Bence-Jones proteins, causing direct tubular cell damage
B. Renal artery thrombosis associated with tubular atrophy
C. Hyperuricemia from urate overproduction or lysis of plasma cells, causing precipitation of urate crystals
D. The suppression of humoral immunity, leading to urinary tract infections that cause chronic tubulointerstitial nephritis
E. Hypercalcemia leading to calcium salt deposition in the kidneys

Key Concept/Objective: To understand the different mechanisms of renal damage in patients with multiple myeloma

Renal insufficiency and acute renal failure are common and significant contributors to morbidity and mortality in patients with multiple myeloma. Clinical indications that a patient with unexplained renal disease has multiple myeloma include (1) low-level proteinuria, as seen on urinary dipstick measurement, in conjunction with high-level proteinuria, as seen on 24-hour quantitative measurement (the dipstick primarily detects albumin, not Bence-Jones protein), (2) a low anion gap (caused by the cationic charge on some monoclonal immunoglobulins), (3) hypercalcemia in the presence of renal failure and a high serum phosphate level, and (4) anemia that is out of proportion to the degree of renal insufficiency. A common manifestation of myeloma is renal insufficiency, present in more than 50% of patients. Excessive production and filtration of monoclonal light chains (Bence-Jones protein) can cause direct tubular cell damage, as well as tubular obstruction by casts. Dysproteinemias can also be associated with tubulointerstitial precipitation of urate crystals, caused by urate overproduction or lysis of plasma cells. In addition, the tubulointerstitial area can be damaged by deposition of calcium salt crystals as a result of hypercalciuria from lytic bone lesions and calciummobilizing humoral factors. Upper and lower urinary tract infections, from suppression of normal humoral immunity, are also common in patients with myeloma and can cause acute and chronic tubulointerstitial damage. (Answer: B—Renal artery thrombosis associated with tubular atrophy)

For more information, see Appel GB: 10 Nephrology: VIII Tubulointerstitial Diseases. ACP Medicine Online (www.acpmedicine.com). Dale DC, Federman DD, Eds. WebMD Inc., New York, January 2004

Chronic Renal Failure and Dialysis

39. A 48-year-old black woman visits your office as a new patient. Her only known medical problems are diabetes and hypertension. She was diagnosed with each illness about 15 years ago. She has no history of kidney disease. She reports her glycosylated hemoglobin (HgA1c) level is typically 7% to 8%, and her average blood pressure is 150/88 mm Hg. Her current drug therapy consists of an angiotensin-converting enzyme (ACE) inhibitor, a calcium channel blocker, and a sulfonylurea. Laboratory studies reveal normocytic anemia, a creatinine level of 2.7 mg/dl, and 3+ protein on urinalysis. Other results are normal.

Which of the following statements regarding chronic renal failure (CRF) is false?

A. The incidence of new cardiovascular disease is the same for those with reduced kidney function as for those with normal kidney function
B. Anemia is directly related to azotemia and is usually evident once the serum creatinine level exceeds 3 mg/dl
C. Like the older preparations, the newer cyclooxygenase-2 (COX-2) nonsteroidal anti-inflammatory drugs (NSAIDs) have an adverse effect on renal function
D. The decline in hematocrit is largely the result of a reduction in the production of erythropoietin by the kidney

Key Concept/Objective: To know the clinical findings associated with CRF and the adverse effect of NSAIDs on renal function

There appears to be a surfeit of cardiovascular disease in persons with impaired kidney function. An analysis by the United States Renal Data System of patients older than 67 years showed that the incidence of new cardiovascular disease was more than 50% greater in those with reduced kidney function, as compared with those having normal kidney function. Anemia is directly related to azotemia and is usually evident once the serum creatinine level exceeds 3 mg/dl. Studies in patients with CRF have shown an inverse correlation of hematocrit with azotemia. This decline in the hematocrit is largely the result of a reduction in the production of erythropoietin by the kidney. Several general therapeutic measures can slow the progressive loss of renal function in CRF. Use of nephrotoxins must be avoided—especially NSAIDs, which may impair renal function because of their effects on prostaglandin synthesis. Patient education on this topic is important, because NSAIDs such as ibuprofen and naproxen are available without prescription. The newer COX-2 NSAIDs also reduce kidney function. (Answer: A—The incidence of new cardiovascular disease is the same for those with reduced kidney function as for those with normal kidney function)

40. A 61-year-old man with progressive hypertensive renal disease visits your office for a routine follow-up visit. You have followed this patient for many years. The patient reports that he has become progressively fatigued over the past few weeks, and his exercise tolerance is failing. He also reports that he has developed persistent, generalized itching. A 24-hour urine collection reveals that his creatinine clearance is stable at 15 ml/min. His blood urea nitrogen (BUN) level is 90 mg/dl, and his creatinine level is 8.5 mg/dl. A nephrologist recently referred the patient to a vascular surgeon for hemodialysis vascular access. He states that his nephrologist has advised that he initiate hemodialysis therapy as soon as his vascular access is placed and matured.

Which of the following statements regarding end-stage renal disease (ESRD) and hemodialysis is false?
A. Infection is second only to cardiovascular disease as a cause of death in patients with ESRD
B. Most deaths caused by infection in patients with ESRD are the result of pneumonia
C. Of the devices for gaining circulatory access, indwelling catheters carry the most risk for infection
D. S. aureus and S. epidermidis are the most commonly identified agents in infections related to dialysis vascular access

Key Concept/Objective: To understand that indwelling vascular catheters are a prominent source of fatal infection in patients with ESRD

ESRD patients are at risk for infection, which is second only to cardiovascular disease as a cause of death in these patients. A recent longitudinal cohort study of ESRD patients identified low serum albumin levels and use of devices such as central venous catheters and artificial arteriovenous fistulas as major risk factors for septicemia. Most deaths caused by infection are related to colonization of devices used to gain temporary access to the circulatory system, such as temporary dialysis catheters; pulmonary and intra-abdominal infections may also occur. Of the devices for gaining circulatory access, indwelling catheters carry the most risk, polytetrafluoroethylene arteriovenous grafts carry somewhat less risk, and native arteriovenous fistulas are least likely to become infected. S. aureus and S. epidermidis are the most commonly identified infectious agents; both may cause septicemia and complications such as endocarditis and septic arthritis. Empirical treatment of either bacteremia or an infection at the site of circulatory access thus requires use of an agent that is effective against staphylococci. (Answer: B—Most deaths caused by infection in patients with ESRD are the result of pneumonia)

41. A 68-year-old man with chronic renal failure secondary to type 2 diabetes mellitus presents with hematemesis. Initial laboratory values indicate a hematocrit of 23%, platelet count of 267,000/mm3, BUN of 126 mg/dl, and creatinine of 10.6 mg/dl. A decision is made to transfuse the patient with 2 units of packed red blood cells and to arrange for upper endoscopy. In addition, preparations are made for temporary access to initiate hemodialysis.

What other therapy would be most likely to minimize bleeding in this patient?

A. Platelet transfusion
B. Desmopressin I.V.
C. Vitamin K I.M.
D. Protamine I.V.
E. Octreotide I.V.

Key Concept/Objective: To be able to recognize the bleeding diathesis of ESRD and understand its management

This patient presents with acute bleeding and compromised renal function. The renal dysfunction has a negative impact on platelet function. Desmopressin appears to help with platelet function. As there is generally an adequate platelet count, platelet transfusion does not have much impact. Vitamin K is used to help reverse problems with the extrinsic clotting cascade. Protamine is used to help reverse the effects of heparin, and octreotide is used for variceal bleeding. Estrogen is also used occasionally for uremic bleeding. (Answer: B—Desmopressin I.V.)

42. A 54-year-old woman with ESRD on hemodialysis presents with a traumatic fracture of the humerus. An orthopedic consultation recommends an open reduction and internal fixation to be performed the following day.

Which of the following medications should most be avoided for this patient?

A. Acetaminophen
B. Morphine sulfate
C. Meperidine
D. Hydromorphone
E. Oxycodone

Key Concept/Objective: To appreciate the need to choose medications carefully for the patient with ESRD

This patient requires postoperative pain management. Most narcotics are metabolized primarily by the liver, but the metabolites of meperidine can accumulate, especially in the setting of compromised renal function. This leads to an increased risk of seizure. Therefore, alternative narcotic (and nonnarcotic) analgesics should be used and the doses monitored closely. (Answer: C—Meperidine)

43. A 58-year-old woman presents to discuss potential hemodialysis. She has had progressive renal failure secondary to polycystic kidney disease and awaits renal transplantation.

Which of the following metabolic abnormalities would most likely be present in this patient?

A. Metabolic acidosis
B. Hypokalemia
C. Hypophosphatemia
D. Hypercalcemia
E. Metabolic alkalosis

Key Concept/Objective: To be able to recognize common metabolic abnormalities in chronic renal failure

A number of metabolic abnormalities can occur in the setting of chronic renal failure. Potassium levels tend to climb because of decreased excretion. Phosphate levels also rise because of the reduction in urine output. The fall in calcium levels is caused by many factors, including decreased intestinal absorption, decreased hydroxylation of vitamin D, increased levels of PTH (with decreased sensitivity), and, at times, decreased intake. Typically, the acid-base disorder is that of metabolic acidosis. This is related to decreased ammonia secretion and inability to excrete titratable acid. There may also be type IV renal tubular acidosis or hyporeninemic hypoaldosteronism (most commonly occurring in diabetic patients). (Answer: A—Metabolic acidosis)

44. A 61-year-old man on chronic hemodialysis undergoes elective hip replacement. In the recovery room, telemetry reveals numerous premature ventricular complexes. An electrocardiogram reveals a prolonged PR interval, a widened QRS complex, and peaking of the T waves.

Which of the following is the most appropriate intravenous intervention for this patient?
A. Magnesium
B. Calcium
C. Potassium
D. Phosphate
E. Heparin

Key Concept/Objective: To be able to recognize hyperkalemia and to understand its association with renal failure

This patient presents with classic electrocardiographic signs of hyperkalemia. The most appropriate first step is the infusion of intravenous calcium, which will help stabilize the myocardium. Other potential interventions include infusions of glucose/insulin, dialysis, albuterol nebulizer, polystyrene resins, and sodium bicarbonate. (Answer: B— Calcium)

45. A 68-year-old woman presents with progressive renal failure secondary to hypertension. On routine laboratory screening, she is found to have a calcium level of 7.8 and an albumin level of 4.2.

Which of the following is contributing to this patient’s calcium level?
A. Enhanced intestinal calcium absorption
B. Low levels of circulating parathyroid hormone (PTH)
C. Hypophosphatemia
D. Decreased vitamin D hydroxylation
E. Decreased fecal calcium

Key Concept/Objective: To be able to recognize the abnormalities in calcium management in the patient with ESRD

Calcium homeostasis is dramatically altered in the patient with ESRD. There is decreased intestinal absorption in the small intestine. The combination of low calcium levels and elevated phosphate levels leads to an increase in PTH. The kidney itself is no longer able to adequately hydroxylate vitamin D. This combination of factors leads to a worsening in bone health. Critical interventions include keeping phosphate levels down, assuring adequate calcium ingestion, and, if necessary, replacing hydroxylated vitamin D. (Answer: D—Decreased vitamin D hydroxylation)

For more information, see Cohen EP: 10 Nephrology: X Chronic Renal Failure and Dialysis. ACP Medicine Online (www.acpmedicine.com). Dale DC, Federman DD, Eds. WebMD Inc., New York, August 2004

Renal Transplantation

46. A 66-year-old man with diabetes mellitus, hypertension, and chronic kidney disease presents for a routine follow-up visit. Despite good control of his hypertension (blood pressure, 124/72 mm Hg) and diabetes (hemoglobin A1c level, 7.0%), the patient’s creatinine level continues to slowly increase (Cr, 3.7 mg/dl). The patient is concerned about the long-term implications of his kidney disease and would like more information regarding his ultimate treatment options.

For this patient, which of the following statements is true?
A. Kidney transplantation results in an improvement in quality of life but a decrease in long-term survival
B. Being older than 65 years precludes this patient from being considered as a kidney transplant recipient
C. Kidney transplant recipients initially have an increase in mortality, but they have an overall improvement in long-term survival
D. Quality of life is similar for dialysis patients and transplantation patients

Key Concept/Objective: To be able to counsel patients who are considering kidney transplantation

Continued improvement in outcomes has made transplantation the treatment of choice for patients with end-stage renal disease. Unlike dialysis patients, transplantation patients have a documented improvement in quality of life and a comparatively high rate of return to employment. A study of more than 220,000 patients showed that long-term survival of patients who received a renal transplant was superior to that of patients who either remained on the transplantation waiting list or continued with long-term dialysis. Patients who underwent transplantation had an initial increase in mortality related to the surgical procedure; however, this initial risk was rapidly eclipsed by the improved long-term survival of transplant recipients. Most transplantation programs offer transplants to medically appropriate recipients regardless of age. Data show that older transplant recipients have excellent survival rates after renal transplantation and may in fact have a lower incidence of episodes of acute rejection than younger recipients. (Answer: C—Kidney transplant recipients initially have an increase in mortality, but they have an overall improvement in long-term survival)

47. The condition of the patient in Question 46 worsens over the next few years, and he undergoes renal transplantation from a living, nonrelated donor. He is started on an immunosuppressant regimen consisting of prednisone, cyclosporine, and mycophenolate mofetil. As his primary care provider, you continue to follow the patient for his hypertension and diabetes, which remain well controlled.

For this patient, which of the following statement is true?
A. The leading cause of death in kidney transplant recipients is opportunistic infection secondary to immunosuppressive therapy
B. Recurrent glomerular kidney disease is the most common cause of graft loss
C. Nephrotoxicity is the most common side effect of mycophenolate mofetil
D. There is a direct correlation between systolic blood pressure and graft half-life; goal systolic blood pressure should be 130 mm Hg or less

Key Concept/Objective: To understand the complications following renal transplantation

The leading cause of death in transplant recipients, as in the general population, is cardiovascular disease. The three most important causes of allograft dysfunction are recurrent glomerular kidney disease, acute rejection, and chronic allograft nephropathy. Although recurrent kidney disease can occur and, in selected cases, may result in progressive loss of renal function, it is much less likely to occur than acute rejection or chronic allograft nephropathy. Long-term studies show that chronic rejection remains the single most important cause of graft loss. Antimetabolites such as mycophenolate mofetil are an important part of immunosuppressive strategies, largely because they have no demonstrable nephrotoxicity and little effect on blood pressure, cholesterol levels, or glycemic control. (Answer: D—There is a direct correlation between systolic blood pressure and graft half-life; goal systolic blood pressure should be 130 mm Hg or less)

48. A 56-year-old woman presents to your clinic for follow-up visit after undergoing renal transplantation 3 months ago. She has been experiencing increasing symptoms of shortness of breath and has had fevers of up to 101° F (38.3° C). You admit her to the hospital and initiate a work-up of her symptoms. Cytomegalovirus (CMV) serologies are positive, and you initiate treatment.

Which of the following interventions could have decreased the likelihood of this patient developing her illness and could have decreased the severity of her illness?
A. Intravenous ganciclovir during pretransplant evaluation
B. Prophylactic ganciclovir at time of transplantation and for 12 weeks thereafter
C. Trimethoprim-sulfamethoxazole daily for 3 to 6 months
D. Amoxicillin-clavulanate, 875 mg p.o., b.i.d.

Key Concept/Objective: To understand the spectrum of infection in the posttransplantation period

Because of immunosuppressive therapy, patients who receive transplants are at risk for acquiring a variety of infections. Infectious agents vary relative to the time of transplantation. The first month after transplantation is characterized by infections that are related to the hospitalization. In this period, urinary tract infections, bacteremia caused by gram-positive cocci, and hospital-acquired pneumonias are common. After the first month and up to 6 months after transplantation, the most common infections are related to immunosuppressive therapy. Opportunistic infections such as CMV, EBV, Pneumocystis carinii infection, and diverse fungal infections predominate. After 6 months, when immunosuppressive therapy is less intense, common infections become prevalent; these include community-acquired pneumonia and cellulitis. CMV is one of the most important posttransplantation infections; it can present as a systemic viral illness, pneumonia, or gastrointestinal disease. Patients can develop primary infection as a result of receiving an organ from a seropositive donor or through reactivation of latent virus. It has been shown that prophylactic oral ganciclovir therapy, started at the time of transplantation and continued for 12 weeks, decreases the incidence and severity of CMV disease. (Answer: B—Prophylactic ganciclovir at time of transplantation and for 12 weeks thereafter)

49. A 47-year-old man who recently received a renal transplant and was started on steroids, cyclosporine, and mycophenolate mofetil presents for routine follow-up. On physical examination, his blood pressure is noted to be 189/96 mm Hg.

Which of the following statements regarding hypertension and renal transplantation is true?

A. Hypertension is a rare posttransplantation complication
B. Mycophenolate mofetil can cause vasoconstriction and worsen hypertension
C. Graft dysfunction causes worsening of hypotension
D. Cyclosporine commonly induces a volume-dependent form of hypertension

Key Concept/Objective: To understand the relationship between immunosuppressive medications and hypertension

With the goal of graft survival in mind, the long-term follow-up of patients undergoing renal transplantation should focus on management of the major causes of morbidity and mortality. Cardiovascular disease, specifically hypertension, is one of the most common posttransplantation complications, affecting 80% to 90% of these patients. The etiology of hypertension in this population is multifactorial but includes diseased native kidneys, use of immunosuppressive medications, graft dysfunction, and, rarely, transplant renal artery stenosis. Although calcineurin inhibitors are the cornerstones of immunosuppression, as a class, these agents commonly cause hypertension. Specifically, cyclosporine causes direct vasoconstriction and induces preglomerular vasoconstriction, resulting in a volume-dependent form of high blood pressure. Other classes of immunosuppressants that cause hypertension are corticosteroids and TOR (target of rapamycin) inhibitors. Antimetabolites, however, such as azathioprine and mycophenolate mofetil, are important in immunosuppressive agents because of their lack of nephrotoxicity and because they have little effect on blood pressure. (Answer: D— Cyclosporine commonly induces a volume-dependent form of hypertension)

50. A 43-year-old woman with end-stage renal disease (ESRD) presents to your clinic for renal transplant evaluation. She has focal segmental glomerular sclerosis and has been doing well for some time on hemodialysis, but she is concerned about “losing the transplanted kidney” because of her original disease.

Which of the following statements regarding recurrence and graft loss associated with her primary renal disease is false?
A. Primary glomerular diseases frequently recur and are commonly associated with graft loss
B. Lupus nephritis rarely recurs after transplantation
C. Type II membranoproliferative glomerulosclerosis has a high recurrence rate, but only one fifth of those patients have graft loss
D. Patients with Alport syndrome can develop anti-glomerular basement membrane (anti-GBM) disease in the allograft

Key Concept/Objective: To understand the risk of disease recurrence in patients with primary glomerular disease

The recurrence rates of different primary renal diseases vary. Primary glomerular diseases frequently recur in the transplanted kidney; however, graft loss secondary to recurrence is uncommon. The patients who are at greatest risk of graft loss are those in whom renal function deteriorated rapidly and aggressively. In these patients, transplantation may be relatively contraindicated. Lupus nephritis, anti-GBM disease, and membranous nephropathy have low recurrence rates and are rarely associated with graft loss. Type II membranoproliferative disease has a high recurrence rate (80% to
90%); however, it too is associated with a low incidence of graft loss. Patients with Alport syndrome can develop anti-GBM disease in the allograft, although this is uncommon, and Alport syndrome is not a contraindication to transplantation. (Answer: A— Primary glomerular diseases frequently recur and are commonly associated with graft loss)

51. A 39-year-old black woman with ESRD secondary to membranous nephropathy presents to your clinic for routine follow-up. She underwent renal transplantatation 3 months ago and is doing well on a regimen of steroids, sirolimus, and cyclosporine. On reviewing results of routine laboratory studies, you note that her total cholesterol level is elevated to 246 mg/dl.

Which of the following statements is true regarding the treatment of this patient’s hyperlipidemia?
A. Her immunosuppresion is unrelated to the elevated cholesterol level
B. Her hyperlipidemia is likely characterized by a low level of low-density lipoprotein (LDL) but a high level of high-density lipoprotein (HDL)
C. Diet and exercise will be sufficient to control her lipid abnormalities
D. Her lipid abnormalities will probably improve within 6 months after transplantation

Key Concept/Objective: To understand the treatment of the patient with coexisting diseases

Of the cardiovascular diseases that affect the renal transplant population, hyperlipidemia is one of the most common. The presence of hyperlipidemia is an important factor for both patient survival and graft survival. If left untreated, arteriosclerosis can occur in the graft as a form of vasculopathy and can lead to tubulointerstitial ischemia, scarring, and fibrosis. Posttransplantation hyperlipidemia is characterized by an increase in total cholesterol and an increase in LDL and very low density lipoprotein levels. Although the etiology of posttransplantation hyperlipidemia is not clear, there is a relationship between hyperlipidemia and the use of immunosuppressive medications, specifically corticosteroids, cyclosporine, and sirolimus. Although the majority of lipid abnormalities resolve within 6 months posttransplantation secondary to a reduction in the doses of immunosuppressant agents, elevations in lipid levels need to be treated aggressively. Usually, dietary measures will not control these lipid abnormalities, and statins are needed for adequate control. Fibrates and nicotinic acid may also be necessary to control refractory lipid levels. In patients with coexisting diabetes, tight glycemic control is essential to good management of hyperlipidemia. (Answer: D—Her lipid abnormalities will probably improve within 6 months after transplantation)

For more information, see Klassen DK, Weir MR: 10 Nephrology: XI Renal Transplantation. ACP Medicine Online (www.acpmedicine.com). Dale DC, Federman DD, Eds. WebMD Inc., New York, December 2004

Benign Prostatic Hyperplasia

52. A 63-year-old man presents to your primary care clinic with a complaint of nocturia. He gets up to urinate two or three times a night and is having trouble getting a restful night’s sleep. Review of symptoms is otherwise negative. Physical examination reveals a large, homogeneous prostate. Laboratory work effectively rules out diabetes mellitus.

Which of the following statements regarding the pathophysiology of benign prostatic hyperplasia (BPH) is true?

A. The hyperplastic process of BPH begins in the peripheral zones and can eventually compress the urethra
B. Only one type of alpha2-adrenergic receptor has been identified in the lower urinary tract
C. Type 1 5a-reductase isoenzyme converts testosterone to dihydrotestosterone preferentially in the prostate
D. Peptide growth factors such as fibroblast growth factors, insulinlike growth factors, and epidermal growth factors are felt to be the local forces that determine prostate growth

Key Concept/Objective: To understand the basic pathophysiology involved in BPH

BPH involves hyperplasia of both the epithelial and the stromal compartments. The hyperplastic process begins in the periurethral and transition zones of the prostate; in contrast, prostate cancer preferentially develops in the peripheral zones. At least three types of alpha2-adrenergic receptors have been identified in the lower urinary tract. The type 1 5a-reductase isoenzyme has low activity in the prostate and is expressed predominantly in the skin and liver. The pathophysiologic mechanisms underlying the development and progression of BPH are incompletely understood. Clearly, BPH involves prolonged exposure of the prostate gland to androgens. In the prostate, interactions between epithelial and stromal cells and the extracellular matrix, mediated primarily by locally produced (intrinsic) growth factors, appear important. These peptide growth factors, which include fibroblast growth factors, insulinlike growth factors, and epidermal growth factors, are felt to be the local forces that determine prostate growth. (Answer: D—Peptide growth factors such as fibroblast growth factors, insulinlike growth factors, and epidermal growth factors are felt to be the local forces that determine prostate growth)

53. A 57-year-old man presents for evaluation of urinary frequency. He has had this problem for 2 years. On review of symptoms, the patient also reports occasional hesitancy and dribbling. Results of physical examination, including digital rectal examination, are normal.

Which of the following statements regarding the diagnosis of BPH is false?
A. Systemic diseases that can mimic BPH include diabetes, heart failure, and hyperparathyroidism
B. It is important to ask about over-the-counter medications because they can contain anticholinergic and sympathomimetic agents that can cause or exacerbate symptoms
C. Abdominal and pelvic ultrasound are indicated in the initial workup of BPH
D. A urinalysis is a part of the workup of BPH to screen for hematuria or infection

Key Concept/Objective: To understand the differential diagnosis and diagnostic workup of BPH

Symptoms of bladder emptying in men with BPH include straining, hesitancy, intermittency, a weak stream, terminal dribbling, and a sensation of incomplete emptying. Bladder filling symptoms include daytime frequency, nocturia, urgency, and urge incontinence. The physician should look for evidence of systemic diseases that can present with lower urinary tract symptoms, particularly urinary frequency and nocturia. Examples of such diseases include diabetes, heart failure, and hyperparathyroidism. Routine tests performed on men with lower urinary tract symptoms should generally include a urinalysis to screen for hematuria and infection. Pyuria suggests infection, either primary or superimposed on bladder outlet obstruction. Microscopic hematuria may indicate simply that the prostate is enlarged and vascular, but it should prompt further evaluation for genitourinary malignancy. Upper urinary tract imaging (by ultrasonography, computed tomography, or intravenous pyelography) and urethrocystoscopy are not indicated for routine cases of lower urinary tract symptoms attributable to BPH. (Answer: C—Abdominal and pelvic ultrasound are indicated in the initial workup of BPH)

54. A patient of yours whom you follow for BPH, hypertension, and osteoarthritis presents to your office.
He has had symptoms of BPH for 3 years now, but over the past 2 to 3 months, his symptoms of hesitancy and straining have worsened to the point that he wishes to pursue therapy.

Which of the following statements regarding the medical management of BPH is true?

A. Alpha1-adrenergic blockers work primarily through relaxation of the detrusor muscle of the bladder
B. Alpha1-adrenergic blockers reduce prostate size and lower prostatespecific antigen (PSA) levels
C. The 5a-reductase inhibitors reduce prostate size and lower PSA levels
D. Alpha blockers offer the same symptom relief as do 5a-reductase inhibitors

Key Concept/Objective: To understand the medical management of BPH

Alpha1-adrenergic blockers work primarily through relaxation of prostatic smooth muscle and relief of the dynamic component of bladder outlet obstruction. However, additional mechanisms have been proposed, including increased apoptosis of prostatic cells. Alpha blockers neither reduce prostate size nor lower PSA levels. Their onset of action is relatively rapid, although most alpha blockers require dose titration to achieve a maximal therapeutic effect while minimizing side effects. The 5a-reductase inhibitors currently available for the treatment of BPH are finasteride and dutasteride. Finasteride selectively and irreversibly binds with the type 2 5a-reductase isoenzyme, which predominates in the prostate and thereby blocks conversion of testosterone to dihydrotestosterone (DHT), the dominant intraprostatic androgen. This agent lowers serum DHT by about 70% and intraprostatic DHT to an even greater degree. Dutasteride is a dual 5a-reductase inhibitor; it blocks both type 1 and type 2 isoenzymes and lowers serum DHT by about 90%. Men who take finasteride at the recommended dose of 5 mg daily or dutasteride at 0.5 mg daily can expect a 20% to 25% reduction in prostate size over the first year of therapy, accompanied by about a 50% reduction in PSA level. (Answer: C—The 5a-reductase inhibitors reduce prostate size and lower PSA levels)

55. A 61-year-old man presents for a follow-up visit for BPH. He has been taking an alpha1-adrenergic blocker for 4 years now; he is currently taking the maximum dose. His symptoms have continued to progress, and he wishes to be referred to a urologist for surgical intervention.

Which of the following statements regarding the surgical treatment of BPH is false?
A. Retrograde ejaculation is a common outcome of transurethral prostatectomy (TURP)
B. Open prostatectomy remains the gold standard for relieving symptoms and reducing the risk of complications for men with BPH
C. Acute urinary retention does not always require surgery and can be managed with bladder rest via catheter drainage
D. Symptom reduction is higher in patients who undergo TURP than in patients placed on watchful waiting

Key Concept/Objective: To understand the surgical options for patients with BPH

TURP remains the gold standard for relieving symptoms and reducing the risk of complications for men with BPH. TURP involves resecting the central adenoma of the hyperplastic prostate transurethrally under direct visualization using a resectoscope with an electrified cutting loop. Retrograde ejaculation is a common outcome of TURP, occurring in the majority of cases. Acute urinary retention used to be considered an absolute indication for surgery. However, small case series have documented that up to half of men with acute retention have a successful voiding trial after a period of bladder rest via catheter drainage, and most of the men who experience success will continue to void, at least over the next 6 months. (Answer: B—Open prostatectomy remains the gold standard for relieving symptoms and reducing the risk of complications for men with BPH)

56. In which of the following patients with symptoms of lower urinary tract dysfunction would it be appropriate to begin an empirical trial of medications for BPH without further testing (other than physical examination)?
A. A 60-year-old man with long-standing hypertension who has nocturia of new onset and intermittent shortness of breath at night
B. A 58-year-old man who complains of weak stream, lower abdominal discomfort, and gross hematuria of 2 months’ duration
C. A 55-year-old man who describes symptoms of weak stream and intermittent straining to urinate, which he has been experiencing for 3 to 4 months
D. An obese 35-year-old man with nocturia of recent onset, daytime urinary frequency, and increased thirst
E. A 72-year-old man with dysuria of 2 days’ duration

Key Concept/Objective: To understand the need to assess for other clinical disorders in patients who present with urinary symptoms similar to BPH

The lower urinary tract symptoms seen in patients with BPH result from bladder outlet obstruction. Typical symptoms of BPH with bladder outlet obstruction are related to impaired bladder emptying (e.g., straining, hesitancy, intermittency, weak stream, terminal dribbling, and incomplete emptying) and bladder irritation/detrussor instability (e.g., daytime frequency, nocturia, urgency, and urge incontinence). In a middle-aged or elderly man with typical symptoms and a confirmatory examination, a presumptive diagnosis of BPH can be made. However, the physician must always be aware of other causes of lower urinary tract symptoms, and an appropriate workup should be carried out. Such causes include congestive heart failure (especially when urinary changes accompany edema, orthopnea, or paroxysmal nocturnal dyspnea), diabetes mellitus, urinary tract infection, and prostatitis. Although patients with BPH may have some hematuria, other diagnoses (including upper urinary tract disease and bladder cancer) should always be ruled out before gross hematuria is attributed to BPH. In a man with dysuria of sudden onset, urinary tract infection or prostatitis is more likely than BPH. (Answer: C—A 55-year-old man who describes symptoms of weak stream and intermittent straining to urinate, which he has been experiencing for 3 to 4 months)

57. A 75-year-old man with a history of mild cognitive impairment is brought to the emergency department because of altered mental status. His wife reports that 1 week ago, he developed nasal congestion and cough, for which he was given an over-the-counter cold medicine/decongestant. Five days ago, he began to complain of difficulty urinating. She states that he was spending an increasing amount of time in the bathroom, yet after leaving the bathroom he would complain that he still had to urinate. He became incontinent of small amounts of blood-tinged urine last night and complained of lower abdominal pain and nausea. On the morning of presentation, he was difficult to arouse and was “not making sense.” Physical examination reveals an ill-appearing elderly man who is disoriented. There is a palpable suprapubic mass that is tender to palpation. Rectal examination reveals a symmetrically enlarged prostate gland. Initial laboratory results include a blood urea nitrogen (BUN) level of 68 and a serum creatinine level of 11 mg/dl. After a Foley catheter is passed with some difficulty, urine output measures approximately 2.0 L within 30 minutes.

Which of the following statements regarding this patient is false?
A. An ultrasound examination of the kidneys and ureters is likely to reveal significant hydronephrosis
B. Sympathomimetic agents such as decongestants may exacerbate obstructive symptoms in patients with BPH
C. Antihistamines with anticholinergic properties may exacerbate obstructive symptoms in patients with BPH and should be avoided
D. The only reasonable approach to managing this patient involves TURP before discharge
E. The patient’s incontinence is likely the result of overflow from an obstructed bladder

Key Concept/Objective: To recognize obstructive uropathy as a potential consequence of BPH and understand the treatment options

This patient has developed severe urinary outflow tract obstruction, resulting in acute renal failure. In patients with BPH, over-the-counter cold and allergy medicines should generally be avoided because the sympathomimetic and anticholinergic agents contained in them can worsen obstructive symptoms. This patient’s severe outflow obstruction resulted in significant urinary retention. With very large bladder volumes, the pressure in the bladder may eventually overcome the resistance at the bladder neck and result in overflow incontinence, as seen in this patient. It is very likely that in this patient, initial upper urinary tract studies would show significant hydronephrosis. It is crucial to recognize such outflow tract obstruction and to relieve it promptly with bladder catheterization, if possible. Acute urinary retention was formerly considered an absolute indication for surgical intervention, but several studies have shown that after a period of bladder rest through catheter drainage combined with medical therapy, up to half of patients will achieve successful voiding. Given the clear precipitating factor involved in the urinary retention seen in this patient, bladder rest and medical therapy with a subsequent voiding trial would be appropriate therapy. (Answer: D—The only reasonable approach to managing this patient involves TURP before discharge)

58. You have been following a 50-year-old man with BPH in clinic for the past 6 months. He had been bothered only slightly by symptoms of mild urinary hesitancy and occasional frequency. Today, however, he complains that since his last visit, his symptoms of straining, hesitancy, dribbling, incomplete emptying, and urinary frequency have been gradually worsening. On review of this patient’s international prostate symptom score (IPSS), you note that his self-reported score of symptoms within the past month totals 15, reflecting moderate symptoms. After further discussion with the patient, you decide to start treatment with terazosin.

Which of the following statements regarding the use of alpha-adrenergic blockers for the treatment of lower urinary tract symptoms of BPH is true?
A. Long-term treatment with alpha-adrenergic blockers has been shown to result in a decrease in prostate size
B. Treatment with alpha-adrenergic blockers can lower the PSA level, thereby altering the cutoff value at which one would be concerned about cancer
C. Alpha-adrenergic blockers induce relaxation of prostatic smooth muscle and may relieve the dynamic component of bladder outlet obstruction
D. Terazosin can usually be initiated at a dose of 10 mg, given before bedtime; a majority of men will achieve notable symptom relief at this dosage
E. Studies have shown that alpha-adrenergic blockers are significantly less effective than the 5a-reductase inhibitor finasteride in reducing BPH symptom scores

Key Concept/Objective: To understand the important aspects of drug therapy for BPH

Alpha-adrenergic blockers are among the mainstays of medical therapy for symptoms of BPH. In the absence of complications, the decision to initiate medical therapy depends in large part on the degree to which the patient is bothered by lower urinary tract symptoms. Symptom scores such as those based on the American Urology Association Symptom Index or IPSS can gauge severity of symptoms and help guide treatment decisions. Alpha-adrenergic blockers work by binding to alpha1-adrenoceptors in the tissue of the bladder neck and reducing neuromuscular tone. They do not, however, have any impact on prostate size or PSA levels (in contrast to finasteride, which lowers PSA levels). Side effects of alpha-adrenergic blockers include orthostatic hypotension, dizziness, and asthenia. Most physicians initiate terazosin at a dose of 1 mg at bedtime for several days to avoid “first-dose” hypotension. The average effective dose of terazosin is closer to 10 mg; the dose should be gradually titrated up to this level in the absence of limiting side effects. Finasteride works by blocking the conversion of testosterone to dihydrotestosterone, which is a mitogen for prostatic tissue growth. Although this therapy may result in an actual decrease in prostate size, the average reduction in symptoms, as reflected in the IPSS, has been much more modest than that achieved with alpha-adrenergic blockers (total score reduction of 2 to 3 points for finasteride, versus 6 to 8 points for alpha-adrenergic blockers). (Answer: C—Alpha-adrenergic blockers induce relaxation of prostatic smooth muscle and may relieve the dynamic component of bladder outlet obstruction)

For more information, see Barry MJ: 10 Nephrology: XIII Benign Prostatic Hyperplasia. ACP Medicine Online (www.acpmedicine.com). Dale DC, Federman DD, Eds. WebMD Inc., New York, September 2004